Sunteți pe pagina 1din 74

Online Prelims TEST - 5 (TEXTBOOK)

( InsightsIAS Mock Test Series for UPSC Preliminary Exam 2020 )

1 The term ‘Lack of Internal democracy’ is sometimes heard in the context of Indian political parties.
The term implies that
1. Power is concentrated at the top and participatory decision-making is largely absent
2. There are no provincial or local branches of the party and workers are drawn from the Central pool
only
3. There is no constitutional document for the party.

Which of the above is/are correct?


A. 1 only
B. 2 and 3 only
C. 1 and 3 only
D. 1, 2 and 3

Correct Answer : A

Answer Justification :

All over the world there is a tendency in political parties towards the concentration of power in one
or few leaders at the top. Parties do not keep membership registers, do not hold organisational
meetings, and do not conduct internal elections regularly.

Ordinary members of the party do not get sufficient information on what happens inside the party.

They do not have the means or the connections needed to influence the decisions. As a result the
leaders assume greater power to make decisions in the name of the party.

Since one or few leaders exercise paramount power in the party, those who disagree with the
leadership find it difficult to continue in the party. More than loyalty to party principles and policies,
personal loyalty to the leader becomes more important.

Q Source: Page 83: Democratic Politics: Class Xth NCERT

2 Consider the following statements about Merchant Discount Rate (alternatively referred to as the
Transaction Discount Rate or TDR).
1. It is the sum total of all the charges and taxes that a digital payment entails.
2. To promote digital transactions, the government will bear MDR charges on certain transactions
made through debit cards, BHIM UPI or Aadhaar-enabled payment systems.
3. It has been capped at 5% by law in India.

Select the correct answer using the codes below.


A. 1 only
B. 1 and 2 only
C. 3 only
D. 1, 2 and 3

Correct Answer : B

1
Online Prelims TEST - 5 (TEXTBOOK)
( InsightsIAS Mock Test Series for UPSC Preliminary Exam 2020 )

Answer Justification :

MDR is a fee charged from a merchant by a bank for accepting payments from customers through
credit and debit cards in their establishments.

MDR compensates the card issuing bank, the lender which puts the PoS terminal and payment
gateways such as Mastercard or Visa for their services.

MDR charges are usually shared in pre-agreed proportion between the bank and a merchant and is
expressed in percentage of transaction amount.

Since 1 January, small merchants pay a maximum MDR of 0.4% of bill value and larger merchants
pay 0.9%.

To promote digital transactions, the government will bear MDR charges on transactions up to Rs
2,000 made through debit cards, BHIM UPI or Aadhaar-enabled payment systems.

Q Source:
https://economictimes.indiatimes.com/wealth/spend/what-is-merchant-discount-rate/articleshow/623
90733.cms?from=mdr

3 Search-lights and vehicle headlights get powerful parallel beams of light by using

A. Convex mirrors
B. Polar mirrors
C. Biofocal mirrors
D. Concave mirrors

Correct Answer : D

Answer Justification :

A ray passing through the principal focus of a concave mirror or a ray which appears to diverge
from the principal focus of a convex mirror, after reflection, will emerge parallel to the principal
axis.

So, concave mirrors are commonly used in torches, search-lights and vehicles headlights to get
powerful parallel beams of light. They are often used as shaving mirrors to see a larger image of the
face. The dentists use concave mirrors to see large images of the teeth of patients.

Large concave mirrors are used to concentrate sunlight to produce heat in solar furnaces.

Q Source: Page 165: Science Xth Standard NCERT

4 Consider the following statements.


1. The public emergency announcement is the highest level of alarm that is only raised during the
gravest of outbreaks.

2
Online Prelims TEST - 5 (TEXTBOOK)
( InsightsIAS Mock Test Series for UPSC Preliminary Exam 2020 )

2. This year, 2019, is the second time in history that WHO has declared a public health emergency.

Which of the above is/are correct?


A. 1 only
B. 2 only
C. Both 1 and 2
D. None

Correct Answer : A

Answer Justification :

Public health emergency of international concern is defined as an “extraordinary event that is


determined to constitute a public health risk to other States through the international spread of
disease and to potentially require a coordinated international response.”

This is the fifth time in history that WHO has declared a public health emergency. The previous
declarations were for the devastating Ebola outbreak in West Africa in 2014-2016 that took lives of
more than 11000 people, spread of Zika virus in Latin America, 2009 Swine flu epidemic and for
polio in 2014. WHO only declares a disease or outbreak a global emergency when it threatens to
affect other countries and requires a coordinated international response.

Ebola virus disease (EVD), formerly known as Ebola haemorrhagic fever, is a severe, often fatal
illness in humans.

Transmission: The virus is transmitted to people from wild animals and spreads in the human
population through human-to-human transmission.

The average EVD case fatality rate is around 50%. Case fatality rates have varied from 25% to 90%
in past outbreaks.

Q Source:
https://www.insightsonindia.com/2019/07/19/insights-daily-current-affairs-pib-19-july-2019/

5 Consider the following statements.


1. A whip in parliamentary parlance is a written order that party members be present for an important
vote, or that they vote only in a particular way.
2. In India, only national/state parties can issue a whip to their members.
3. The anti-defection law allows the Speaker/Chairperson to disqualify such a member who goes
against the voting directions of the whip.

Select the correct answer using the codes below.


A. 1 and 2 only
B. 2 and 3 only
C. 1 and 3 only
D. 1, 2 and 3

3
Online Prelims TEST - 5 (TEXTBOOK)
( InsightsIAS Mock Test Series for UPSC Preliminary Exam 2020 )

Correct Answer : C

Answer Justification :

The term is derived from the old British practice of “whipping in” lawmakers to follow the party
line.

Parties appoint a senior member from among their House contingents to issue whips — this member
is called a Chief Whip, and he/she is assisted by additional Whips.

Any registered party can issue a whip.

In India, rebelling against a three-line whip can put a lawmaker’s membership of the House at risk.

The anti-defection law allows the Speaker/Chairperson to disqualify such a member; the only
exception is when more than a third of legislators vote against a directive, effectively splitting the
party.

Importance of whips in our political system:

In the parliamentary form of Government, Whips of various political parties are the vital links of the
internal organization of parties, inside the legislatures. The efficient and smooth functioning of
Parliament and State Legislatures depends, to a considerable extent, upon the office of the Whip.
The Whips can be rightly said to be the managers of the parties within the legislatures.

Q Source:
https://www.insightsonindia.com/2019/07/19/insights-daily-current-affairs-pib-19-july-2019/

6 The prime objectives of the SAGAR MAITRI Mission are


1. Data collection from the entire North Indian Ocean
2. Enable construction of ship bases in the Indian Ocean to be used mutually by members of the
accord

Which of the above is/are correct?


A. 1 only
B. 2 only
C. Both 1 and 2
D. None

Correct Answer : A

Answer Justification :

Justification: SAGAR MAITRI is a unique initiative of DRDO which aligns with the broad objective
of Prime Minister Shri Narendra Modi’s policy declaration “Safety And Growth for All in the Region
(SAGAR)” to promote closer co-operation in socio-economic aspects as well as greater scientific
interaction especially in ocean research among Indian Ocean Rim (IOR) countries.
4
Online Prelims TEST - 5 (TEXTBOOK)
( InsightsIAS Mock Test Series for UPSC Preliminary Exam 2020 )

Under the aegis of PM’s policy, specific scientific component of DRDO is “MAITRI (Marine & Allied
Interdisciplinary Training and Research Initiative)”.

SAGAR MAITRI Mission-2 commemorates the Golden Jubilee Celebrations of India’s lone research
ship INS Kistna’s missions as part of the historic International Indian Ocean Expeditions (IIOE),
which took place during 1962-65.

As part of the mission, INS Sagardhwani will revisit the selected tracks of INS Kistna and provide
NPOL scientists ample opportunities to collaborate and garner a close working relationship with the
oceanographic counterparts of the IOR countries.

The prime objectives of the SAGAR MAITRI Mission are data collection from the entire North Indian
Ocean, focussing on the Andaman Sea and adjoining seas and establishing long-term collaboration
with eight IOR countries in the field of ocean research and development.

The programme also aims at establishing long term scientific collaboration with these countries in
the field of ‘Ocean Research & Development’ and data collection with a focus in the Andaman Sea.

Q Source:
https://www.insightsonindia.com/2019/07/19/insights-daily-current-affairs-pib-19-july-2019/

7 Rock paintings have been discovered in which of these places?


1. Lakhudiyar
2. Kumaon hills
3. Lenyadri Caves

Select the correct answer using the codes below.


A. 1 and 2 only
B. 1, 2 and 3
C. 1 and 3 only
D. 2 only

Correct Answer : B

Answer Justification :

Justification: Statement 1 and 2: Remnants of rock paintings have been found on the walls of the
caves situated in several districts of Madhya Pradesh, Uttar Pradesh, Andhra Pradesh, Karnataka
and Bihar.

Some paintings have been reported from the Kumaon hills in Uttarakhand also. The rock shelters on
banks of the River Suyal at Lakhudiyar, about twenty kilometres on the Almora– Barechina road,
bear these prehistoric paintings.

Lakhudiyar literally means one lakh caves. The paintings here can be divided into three categories:
man, animal and geometric patterns in white, black and red ochre. Humans
are represented in stick-like forms.

5
Online Prelims TEST - 5 (TEXTBOOK)
( InsightsIAS Mock Test Series for UPSC Preliminary Exam 2020 )

Statement 3: This is one of the hidden paradises, when it comes to cave painting. Out of 40 different
caves of this cave group, several caves have unique Buddhist paintings. These caves of Maharashtra
served as thriving Buddhist temples up to 3rd century BC. During 19th century, several caves were
converted into Hindu caves. These caves have simple styled paintings with traces of Buddhist
paintings.

Q Source: Ch2: Introduction to Indian arts: Class XI

8 The countries with which Russia shares its land border does NOT include

A. North Korea
B. Finland
C. Poland
D. Romania

Correct Answer : D

Answer Justification :

Learning: These nations which share border with Russia are North Korea, China, Norway, Finland,
Ukraine, Kazakhstan, Poland, Georgia, Mongolia, Latvia, Estonia, Azerbaijan, Belarus, and
Lithuania.

Q Source: Map based: Northern Asia

6
Online Prelims TEST - 5 (TEXTBOOK)
( InsightsIAS Mock Test Series for UPSC Preliminary Exam 2020 )

9 Which of the following herbs are used to produce bio-preservatives?


1. Cloves
2. Vanilla
3. Ginger
4. Onion

Select the correct answer using the codes below.


A. 1, 2, 3 and 4
B. 2 and 4 only
C. 1 and 3 only
D. 2, 3 and 4 only

Correct Answer : A

Answer Justification :

Justification: In today's food industry, chemical preservatives are commonly used to maintain raw
material quality, to ensure safety and stability of the products during their extended shelf life. But
consumers world over prefer food products that are natural, additive free, have less salt and
acceptable shelf life with assured quality. However, the use of chemical preservatives to inhibit
growth of spoilage bacteria in modem food systems has been recognized as deterrent to health.
Thus this has necessitated the need to exploit the natural biological substance from plants, animals
and microorganisms, which are safe for human health and are capable of producing substances that
act as preservatives, called 'Biopreservatives'.

Q Source: http://agritech.tnau.ac.in/org_farm/orgfarm_miscellaneous.html#maha

10 Which of the following best describes a ‘ploonet’?

A. An unhabitable zone on a planet


B. Moon ejected out of orbit
C. An exoplanet in habitable zone

7
Online Prelims TEST - 5 (TEXTBOOK)
( InsightsIAS Mock Test Series for UPSC Preliminary Exam 2020 )

D. The largest satellite moon of a planet

Correct Answer : B

Answer Justification :

Learning: These are excerpts from an Indian express article:

“What if a moon went rogue and acted like a planet? It can happen, astronomers say, and give it a
name.

Imagine an exoplanet, a gas giant orbiting its star while a moon orbits the planet itself. Now
suppose the moon turns rogue as it moves nearer to its star, breaking away — or being forced out of
its orbit by the planet — and going off on its own trip, in effect behaving like a planet in its own
right. What is it now: a moon or a planet? Call it “ploonet”, suggests an international team of
astronomers.

They modelled the likely behaviour of giant exomoons predicted to form around massive planets —
and found that they would be expelled and sent packing, they say in a statement released by
Australia’s Macquarie University.

Roughly 50% of these ejected moons would survive the immediate expulsion and avoid any
subsequent collision with the planet or the star, ending up in orbit around the star, but in eccentric
orbits. If such a scenario does take place, these “ploonets”, could potentially explain several
puzzling phenomena.

Why have astronomers not yet confirmed the existence of a single exomoon, when they found over
4,000 exoplanets? Have the exomoons turned into ploonets?

In a paper to be published in the journal Monthly Notices of the Royal Astronomical Society, the
researchers explain that the angular momentum between the planet and its moon results in the
moon escaping the gravitational pull of its parent. They concede, however, that ploonets remain
hypothetical.”

Q Source:
https://indianexpress.com/article/explained/ploonet-what-if-a-moon-went-rogue-and-acted-like-a-plan
et-5834863/

11 Why are floods so destructive in Assam?


1. Brahmaputra river carries a large amount of silt.
2. Sudden change in velocity of Brahmaputra as it enters India allows it to deposit its load which
causes river overflows.

Which of the above is/are correct?


A. 1 only
B. 2 only
C. Both 1 and 2

8
Online Prelims TEST - 5 (TEXTBOOK)
( InsightsIAS Mock Test Series for UPSC Preliminary Exam 2020 )

D. None

Correct Answer : C

Answer Justification :

Justification: Apart from incessant rainfall during the monsoon, there are many contributory
factors, natural and man-made.

At the crux is the very nature of the river Brahmaputra —dynamic and unstable. Its 580,000
sq km basin spreads over four countries: China, India, Bangladesh and Bhutan, with diverse
environments.
The Brahmaputra features among the world’s top five rivers in terms of discharge as well as
the sediment it brings.
The vast amount of sediment comes from Tibet, where the river originates. That region is
cold, arid and lacks plantation. Glaciers melt, soil erodes and all of it results in a highly
sedimented river.
By the time the river enters Assam — a state comprising primarily floodplains surrounded by
hills on all sides — it deposits vast amounts of this silt, leading to erosion and floods. As the
river comes from a high slope to a flat plain, its velocity decreases suddenly and this results in
the river unloading the sediment. The river’s channels prove inadequate amid this siltation,
leading to floods.
Again, because of the earthquake-prone nature of the region, the river has not been able to
acquire a stable character. Following the devastating earthquake of 1950, the level of the
Brahmaputra rose by two metres in Dibrugarh area in eastern Assam.
Besides these natural factors are the man-made ones — habitation, deforestation, population
growth in catchment areas (including in China) — which lead to higher sedimentation. For
example, the sediment deposition itself creates temporary sandbars or river islands.
It is common for people to settle in such places, which restricts the space the river has to
flow. When rainfall is heavy, it combines with all these factors and leads to destructive floods.
This happens very frequently.

Q Source:
https://www.insightsonindia.com/2019/07/19/insights-daily-current-affairs-pib-19-july-2019/

12 Consider the following statements.


Assertion (A): The energy pyramid of an ecosystem is always upright and narrows to the top.
Reason (R): The biomass in the upper trophic levels is generally very high as compared to the lower
trophic levels.

In the context of the above, which of these is correct?


A. A is correct, and R is an appropriate explanation of A.
B. A is correct, but R is not an appropriate explanation of A.
C. A is correct, but R is incorrect.
D. Both A and R are incorrect.

9
Online Prelims TEST - 5 (TEXTBOOK)
( InsightsIAS Mock Test Series for UPSC Preliminary Exam 2020 )

Correct Answer : C

Answer Justification :

Justification: The green plants in a terrestrial ecosystem capture about 1% of the energy of
sunlight that falls on their leaves and convert it into food energy.

When green plants are eaten by primary consumers, a great deal of energy is lost as heat to the
environment, some amount goes into digestion and in doing work and the rest goes towards growth
and reproduction.

An average of 10% of the food eaten is turned into its own body and made available for the next
level of consumers.

Since each higher trophic level receives only a fraction of energy of the lower trophic levels, the
energy pyramid is narrow at the top. So, A is correct.

But, generally (barring some aquatic ecosystems) lower trophic levels have higher biomass as
compared to the higher trophic levels. So, R is incorrect. Moreover, even if this were to be so, it
doesn’t explain A.

Q Source: Page 261: Science Xth Standard NCERT

13 Which of the following was NOT one of the criteria used by 11th-14th Finance Commission to
recommend allocation of resources to the states?

A. Fiscal discipline of the state


B. Forest Cover of the state
C. Area of the State
D. Poverty rate in the state

Correct Answer : D

Answer Justification :

Justification: Statement 1:

10
Online Prelims TEST - 5 (TEXTBOOK)
( InsightsIAS Mock Test Series for UPSC Preliminary Exam 2020 )

The Finance Commission is appointed by the President under Article 280 of the Constitution.

Besides the taxes devolved to states, another source of transfers from the centre to states is grants-
in-aid. As per the recommendations of the 14th Finance Commission, grants-in-aid constitute 12% of
the central transfers to states. The 14th Finance Commission had recommended grants to states for
three purposes: (i) disaster relief, (ii) local bodies, and (iii) revenue deficit.

Q Source:
https://www.insightsonindia.com/2019/07/18/insights-daily-current-affairs-pib-18-july-2019/

14 Kulhs are water channels meant for carrying water from glaciers to villages in mountain valleys.
These Kulhs are traditionally found in

A. Himachal Pradesh
B. Ladakh
C. Assam
D. Nagaland

Correct Answer : A

Answer Justification :

Learning: Parts of Himachal Pradesh had evolved a local system of canal irrigation called kulhs
over four hundred years ago. The water flowing in the streams was diverted into man-made

11
Online Prelims TEST - 5 (TEXTBOOK)
( InsightsIAS Mock Test Series for UPSC Preliminary Exam 2020 )

channels which took this water to numerous villages down the hillside.

The management of the water flowing in these kulhs was by common agreement among all the
villages. Interestingly, during the planting season, water was first used by the village farthest away
from the source of the kulh, then by villages progressively higher up. These kulhs were managed by
two or three people who were paid by the villagers.

In addition to irrigation, water from these kulhs also percolated into the soil and fed prings at
various points. After the kulhs were taken over by the Irrigation Department, most of them became
defunct and there is no amicable sharing of water as before.

Q Source: Page 274: Science Xth Standard NCERT

15 The practice of Vegetative Propagation is beneficial becauseIt can help propagate even those plants
that have lost the capacity to produce seeds.
1. It can help propagate even those plants that have lost the capacity to produce seeds.
2. Plants produced under the practice bear no genetic resemblance to the parent plant thus promoting
biodiversity.

Which of the above is/are correct?


A. 1 only
B. 2 only
C. Both 1 and 2
D. None

Correct Answer : A

Answer Justification :

Justification & Learning: There are many plants in which parts like the root, stem and leaves
develop into new plants under appropriate conditions.

Unlike in most animals, plants can indeed use such a mode for reproduction. This property of
vegetative propagation is used in methods such as layering or grafting to grow many plants
like sugarcane, roses, or grapes for agricultural purposes.
Plants raised by vegetative propagation can bear flowers and fruits earlier than those
produced from seeds. Such methods also make possible the propagation of plants such as
banana, orange, rose and jasmine that have lost the capacity to produce seeds. So, 1 is
correct.
Another advantage of vegetative propagation is that all plants produced are genetically
similar enough to the parent plant to have all its characteristics. So, 2 is wrong.

Q Source: Page 131: Science Xth Standard NCERT

16 Consider the following statements.


1. The first discovery of rock paintings was made in India around the middle of 20th CE by an

12
Online Prelims TEST - 5 (TEXTBOOK)
( InsightsIAS Mock Test Series for UPSC Preliminary Exam 2020 )

archaeologist, Archibold Carlleyle.


2. Anderson, Mitra and Ghosh were some of the early archaeologists who discovered a large number
of sites in the Indian sub-continent.

Select the correct answer using the codes below.


A. 1 only
B. 2 only
C. Both 1 and 2
D. None of the above

Correct Answer : B

Answer Justification :

Justification: It is interesting to know that the first discovery of rock paintings was made in India
in 1867–68 (19th CE) by an archaeologist, Archibold Carlleyle, twelve years before the discovery of
Altamira in Spain. Cockburn, Anderson, Mitra and Ghosh were the early archaeologists who
discovered a large number of sites in the Indian sub-continent.

Remnants of rock paintings have been found on the walls of the caves situated in several districts of
Madhya Pradesh, Uttar Pradesh, Andhra Pradesh, Karnataka and Bihar. Some paintings have been
reported from the Kumaon hills in Uttarakhand also.

Q Source: Ch1: Introduction to Indian arts: Class XI

17 Consider the following statements.


1. The same person cannot be the Governor of two or more states.
2. There is no provision of impeachment for a Governor, as in the case of the President.
3. Similar to the President, the Governor enjoys diplomatic and military powers.

Select the correct answer using the codes below.


A. 1 and 3 only
B. 2 only
C. 1, 2 and 3
D. 2 and 3 only

Correct Answer : B

Answer Justification :

Justification: Governor is the nominal head of a state, unlike the Chief Minister who is the real
head of a state in India.

According to an amendment in the Constitution of India (7th Constitutional Amendment Act),


brought about in 1956, the same person can be the Governor of two or more states.

13
Online Prelims TEST - 5 (TEXTBOOK)
( InsightsIAS Mock Test Series for UPSC Preliminary Exam 2020 )

Appointment and removal:

The governors and lieutenant-governors are appointed by the president for a term of 5 years.

The term of governor’s office is normally 5 years but it can be terminated earlier by: Dismissal by
the president on the advice of the prime minister of the country, at whose pleasure the governor
holds office or Resignation by the governor. Thus, the term is subject to pleasure of the president.

There is no provision of impeachment, as it happens for the president.

Article 157 and Article 158of the Constitution of India specify eligibility requirements for the post of
governor.

Powers:

Like the President of India, the Governor of any state in India is vested with certain executive,
legislative and judicial powers.

He or she also possesses certain discretionary or emergency powers.

But one major difference in the powers enjoyed by the President and those enjoyed by the
Governor is, the Governor does not have any diplomatic or military powers. More on this will
be covered in later tests.

Q Source:
https://www.insightsonindia.com/2019/07/17/insights-daily-current-affairs-pib-17-july-2019/

18 The Arctic Circle


1. passes through all the Scandinavian countries
2. does not pass through the city of Moscow in Russia

Select the correct answer using the codes below.


A. 1 only
B. 2 only
C. Both 1 and 2
D. None of the above

Correct Answer : C

Answer Justification :

Justification:

14
Online Prelims TEST - 5 (TEXTBOOK)
( InsightsIAS Mock Test Series for UPSC Preliminary Exam 2020 )

Q Source: Northern Asia: Map

19 The Motor Vehicles (Amendment) Bill, 2019 seeks to amend the Motor Vehicles Act, 1988 to provide
for road safety. It addresses which of the following sub-sectors/issues?
1. Cab aggregators
2. Compulsory electric/hybrid vehicles from 2025
3. Protection of Samaritans
4. Establishment of a Road Safety Board

Select the correct answer using the codes below.


A. 1, 2, 3 and 4
B. 2, 3 and 4 only
C. 1 and 2 only
D. 1, 3 and 4 only

Correct Answer : D

Answer Justification :

Justification: Statement 1: The Bill defines aggregators as digital intermediaries or market places
which can be used by passengers to connect with a driver for transportation purposes (taxi
services). These aggregators will be issued licenses by state. Further, they must comply with the
Information Technology Act, 2000.

Statement 2: No such provision. But, there is a transportation policy.

15
Online Prelims TEST - 5 (TEXTBOOK)
( InsightsIAS Mock Test Series for UPSC Preliminary Exam 2020 )

The central government may develop a National Transportation Policy, in consultation with state
governments. The Policy will: (i) establish a planning framework for road transport, (ii) develop a
framework for grant of permits, and (iii) specify priorities for the transport system, among other
things.

Statement 3: The Bill defines a good samaritan as a person who renders emergency medical or non-
medical assistance to a victim at the scene of an accident. The assistance must have been (i) in good
faith, (ii) voluntary, and (iii) without the expectation of any reward. Such a person will not be liable
for any civil or criminal action for any injury to or death of an accident victim, caused due to their
negligence in providing assistance to the victim.

Statement 4: The Bill provides for a National Road Safety Board, to be created by the central
government through a notification. The Board will advise the central and state governments on all
aspects of road safety and traffic management .

You can read other details in the link provided below:

Q Source:
https://www.insightsonindia.com/2019/07/17/insights-daily-current-affairs-pib-17-july-2019/

20 Consider the following about Central Asian resources.


1. Central Asia is a major producer of Uranium, gas and oil.
2. No Central Asian country has Gold reserves.
3. Most Central Asian countries are major importers of cotton and Aluminum.

Select the correct answer using the codes below.


A. 1 only
B. 1 and 3 only
C. 2 and 3 only
D. 1, 2 and 3

Correct Answer : A

Answer Justification :

Justification: Statement 1: Kazakhstan and Turkmenistan have abundant oil and natural gas
reserves and Uzbekistan’s own reserves make it more or less self-sufficient.

Statement 2:Kyrgyzstan, Tajikistan and Uzbekistan all have gold reserves and Kazakhstan has the
world’s largest uranium reserves.

Statement 3:Fluctuating global demand for cotton, aluminium and other metals (except gold) in
recent years has hit Tajikistan hardest, since aluminium and raw cotton are its chief exports − the
Tajik Aluminium Company is the country’s primary industrial asset.

In fact in 2014, it announced the government’s intention to reduce the acreage of land
cultivated by cotton to make way for other crops.
16
Online Prelims TEST - 5 (TEXTBOOK)
( InsightsIAS Mock Test Series for UPSC Preliminary Exam 2020 )

Uzbekistan and Turkmenistan are major cotton exporters themselves,

Q Source: Map based: Central Asia

21 What will be the benefit(s) of the implementation of the Crime and Criminal Tracking Network and
Systems (CCTNS) project?
1. It would lead to a Central citizen portal having linkages with State level citizen portals that will
provide a number of citizen friendly services like Police Verification for various purposes.
2. It will enable Pan-India criminal/accused name search in the regional language for improved inter-
state tracking of criminal movement.

Select the correct answer using the codes below.


A. 1 only
B. 2 only
C. Both 1 and 2
D. None of the above

Correct Answer : C

Answer Justification :

Justification: Crime and Criminal Tracking Network and Systems (CCTNS) is a project initiated in
June 2009 which aims at creating a comprehensive and integrated system for enhancing the
efficiency and effectiveness of policing at the Police Station level. This will be done through
adoption of principles of e-Governance, and creation of a nationwide networked infrastructure for
evolution of IT-enabled state-of-the-art tracking system around “investigation of crime and detection
of criminals”. CCTNS is a Mission Mode Project (MMP) under the National e-Governance Plan of
Govt. of India.

The Project will interconnect about 15000 Police Stations and additional 5000 offices of supervisory
police officers across the country and digitize data related to FIR registration, investigation and
charge sheets in all Police Stations.

It will not only automate Police functions at Police station and higher levels but will also create
facilities and mechanism to provide public services like registration of online complaints,
ascertaining the status of case registered at the police station, verification of persons etc.

In 2015, an additional objective of establishing a basic platform for an Inter-operable Criminal


Justice System (ICJS) was added to the Project.

Statement 1: The Full implementation of the Project with all the new components would lead to a
Central citizen portal having linkages with State level citizen portals that will provide a number of
citizen friendly services like Police Verification for various purposes including passport verification,
reporting a crime including cyber-crime and online tracking of the case progress etc.

Statement 2: The project will enable National level crime analytics to be published at increased

17
Online Prelims TEST - 5 (TEXTBOOK)
( InsightsIAS Mock Test Series for UPSC Preliminary Exam 2020 )

frequency, which will help the policy makers as well as lawmakers in taking appropriate and timely
action, it will also enable Pan-India criminal/accused name search in the regional language for
improved inter-state tracking of criminal movement. This would lead to development of a national
database of crimes and criminals.

Q Source:
https://www.insightsonindia.com/2019/07/17/insights-daily-current-affairs-pib-17-july-2019/

22 Which of the following is/are the key features of Indian Federalism?


1. The existence and authority of each tier of government is constitutionally guaranteed.
2. Division of powers between State and Centre cannot be arbitrarily manipulated by the Central
government alone.

Select the correct answer using the codes below.


A. 1 only
B. 2 only
C. Both 1 and 2
D. None of the above

Correct Answer : C

Answer Justification :

Justification: Each tier has its own jurisdiction in specific matters of legislation, taxation and
administration; for e.g. in India states legislate in matters of police, and Centre legislates in areas of
national security as a whole.

Tiers may be dependent upon each other; however not completely, else it erodes autonomy.

The fundamental provisions of the constitution cannot be unilaterally changed by one level of
government. Such changes require the consent of both the levels of government.

Q Source: Revision: Democratic Politics NCERT

23 With reference to The Thirty Meter Telescope (TMT) in news, consider the following statements.
1. It has been designed for near-ultraviolet to mid-infrared observations.
2. It is an international project being partly funded by India.
3. It will enable scientists to study gravitational waves from massive objects far away.

Select the correct answer using the codes below.


A. 1, 2 and 3
B. 2 and 3 only
C. 1 and 2 only
D. 3 only

18
Online Prelims TEST - 5 (TEXTBOOK)
( InsightsIAS Mock Test Series for UPSC Preliminary Exam 2020 )

Correct Answer : C

Answer Justification :

Justification: It is a proposed astronomical observatory with an extremely large telescope (ELT).

It is an international project being funded by scientific organisations of Canada, China, India, Japan
and USA.

Planned location: Mauna Kea on the island of Hawaii in the US state of Hawaii.

The TMT is designed for near-ultraviolet to mid-infrared observations, featuring adaptive optics to
assist in correcting image blur.

TMT will enable scientists to study fainter objects far away from us in the Universe, which gives
information about early stages of evolution of the Universe.

It will give us finer details of not-so-far-away objects like undiscovered planets and other objects in
the Solar System and planets around other stars.

Q Source: More details here


https://www.insightsonindia.com/2019/07/16/insights-daily-current-affairs-pib-16-july-2019/

24 Which of the following can be classified as forms of Gender based violence in India?
1. Dowry
2. Child Marriage
3. Sex selective abortion

Select the correct answer using the codes below.


A. 2 only
B. 3 only
C. 1 and 2 only
D. 1, 2 and 3

Correct Answer : D

Answer Justification :

Justification: In India, gender based violence has many manifestations; from the more universally
prevalent forms of domestic and sexual violence including rape, to harmful practices such as,
dowry, honour killings, acid attacks, witch – hunting, sexual harassment, child sexual abuse,
trafficking for commercial sexual exploitation, child marriage, sex selective abortion, sati etc.

Gender Based Violence (GBV) is a global health, human rights and development issue that
transcends geography, class, culture, age, race and religion to affect every community and country
in every corner of the world.

The Article 1 of UN Declaration on the Elimination of Violence 1993 provides a definition of gender

19
Online Prelims TEST - 5 (TEXTBOOK)
( InsightsIAS Mock Test Series for UPSC Preliminary Exam 2020 )

– based abuse, calling it “any act of gender – based violence that results in, or is likely to result in,
physical, sexual or psychological harm or suffering to women, including threats of such acts,
coercion or arbitrary deprivation of liberty, whether occurring in public or in private life”.

Q Source:
https://www.insightsonindia.com/2019/07/19/insights-daily-current-affairs-pib-19-july-2019/

25 In the human body significant magnetic field is produced by which of these organs?

A. Spleen and liver


B. Heart and Brain
C. Stomach and Duodenum
D. Hands and legs

Correct Answer : B

Answer Justification :

Learning: An electric current always produces a magnetic field. Even weak ion currents that travel
along the nerve cells in our body produce magnetic fields.

When we touch something, our nerves carry an electric impulse to the muscles we need to
use. This impulse produces a temporary magnetic field.
These fields are very weak and are about one-billionth of the earth’s magnetic field. Two main
organs in the human body where the magnetic field produced is significant, are the heart and
the brain. The magnetic field inside the body forms the basis of obtaining the images of
different body parts.
This is done using a technique called Magnetic Resonance Imaging (MRI). Analysis of these
images helps in medical diagnosis.

Q Source: Page 232: Science Xth Standard NCERT

26 Spent pot lining (SPL), which contains high level of cyanide and fluoride and is carcinogenic in
nature, is mainly released from which of these industries?

A. Tanning
B. Hydrocarbon distillation
C. Aluminium
D. Plastic recycling

Correct Answer : C

Answer Justification :

Justification: Spent pot lining (SPL) is a carbonaceous waste from primary aluminum production

20
Online Prelims TEST - 5 (TEXTBOOK)
( InsightsIAS Mock Test Series for UPSC Preliminary Exam 2020 )

which could potentially participate in the ironmaking with consolidated raw materials.

Aluminum smelters have long wrestled with the problem of recycling and disposal of spent pot
lining. This hazardous waste stream contains toxic compounds including leachable fluorides and
cyanides plus it reacts with water producing explosive gas mixtures of hydrogen and methane. If
land filled it can cause serious environmental damage and is a major threat to human health.
Historic landfill sites have been “cleaned-up” by authorities who have spent hundreds of millions
attempting to remediate these toxic time bombs.

It is carcinogenic in nature and must be scientifically utilised or detoxified.

It has also been classified as hazardous waste under the Schedule to the Hazardous Waste
Management Rules, 2016.

Q Source:
https://www.insightsonindia.com/2019/07/16/insights-daily-current-affairs-pib-16-july-2019/

27 Which of the following is/are benefits of Probiotics?


1. They increase the pH of the stomach making digestion easier.
2. They make diarrheal diseases milder.
3. They kill major bacterial strains in the human gut improving bowel movement.

Select the correct answer using the codes below.


A. 1 and 2 only
B. 3 only
C. 2 only
D. 1 and 3 only

Correct Answer : C

Answer Justification :

Justification: They are live food supplements used in yoghurt and other fermented milk products.

It includes Lactobacillus acidophillus and Bifidobacterium bifidum. A minimum of 10^8 bacteria per
1 ml must get to the colon alive to have any significant effect.

These bacteria improve the microbial spectrum in the gut and thus contribute to the following
effects:

Influence immunity and hence prevent or make diarrheal diseases milder


Decrease the risk of colon cancer
Decrease cholesterol absorption
Produce acids that decrease the pH in the gut and thus increase the absorption of minerals
such as calcium and phosphorous.

Q Source: AR: Past year UPSC papers

21
Online Prelims TEST - 5 (TEXTBOOK)
( InsightsIAS Mock Test Series for UPSC Preliminary Exam 2020 )

28 Nettle leaves have stinging hair, which cause painful stings when touched accidentally. Despite
giving pain, stinging nettle is useful as?

A. Analgesic
B. Antioxidant
C. Antimicrobial agent
D. All of the above

Correct Answer : D

Answer Justification :

Learning: Stinging nettle contains a number of chemicals, such as serotonin, histamine and
acetylcholine, some of which can be very irritating.

According to the University of Maryland Medical Center, the plant has been used most commonly
throughout history as a diuretic and for treating painful muscles and joints, eczema, arthritis, gout,
and anemia. Today, it’s used primarily to treat urinary issues, as well as allergies and joint pain.

Studies have shown stinging nettle has antioxidant, antimicrobial, anti-ulcer, astringent and
analgesic capabilities

Q Source: AR: Page 28: Science Xth Standard NCERT

29 GAFA tax recently seen in news is a kind of

A. Corporation tax
B. Ecological cess
C. Inter-generational equity tax
D. Personal tax

Correct Answer : A

Answer Justification :

Justification: It is a legislation — dubbed the GAFA tax — an acronym for Google, Apple, Facebook
and Amazon – passed recently by France Parliament.

With this, France has become the first major economy to impose a tax on digital giants.

The new law aims at plugging a taxation gap that has seen some internet heavyweights paying next
to nothing in countries where they make huge profits.

The law will levy a 3 per cent tax on total annual revenues of the largest tech firms providing
services to French consumers.

22
Online Prelims TEST - 5 (TEXTBOOK)
( InsightsIAS Mock Test Series for UPSC Preliminary Exam 2020 )

Q Source:
https://www.insightsonindia.com/2019/07/13/insights-daily-current-affairs-pib-13-july-2019/

30 The Shramaṇ tradition retained their distinct identity from Hinduism by


1. Rejecting the authority of the Vedas as the highest source of knowledge
2. Repudiating concepts like karma, rebirth and consciousness

Select the correct answer using the codes below.


A. 1 only
B. 2 only
C. Both 1 and 2
D. None of the above

Correct Answer : A

Answer Justification :

Justification: Statement 1: The śramaṇa refers to a variety of austerity, ascetic traditions from the
middle of the 1st millennium BCE. The śramaṇas were individual, experiential and free-form
traditions.

Part of the śramaṇa tradition retained their distinct identity from Hinduism by rejecting the
epistemic (source of knowledge) authority of the Vedas, while a part of the śramaṇa tradition
became part of Hinduism as one stage in the Ashrama dharma, that is as it renunciate sannyasins.

During its later semantic development, the term came to refer to several non-Brahmanical ascetic
movements parallel to but separate from the Vedic religion. The śramaṇa tradition includes Jainism,
Buddhism, and others such as the Ājīvikas, Ajñanas and Cārvākas.

Statement 2: The Śramaṇic traditions have a diverse range of beliefs, ranging from accepting or
denying the concept of soul, consciousness, fatalism to free will, idealization of extreme asceticism
to that of family life, wearing dress to complete nudity in daily social life, strict ahimsa (non-
violence) and vegetarianism to permissibility of violence and meat-eating.

Q Source: AR: Ch3: Introduction to Indian arts: Class XI

31 Which of the following Central Asian countries border China?


1. Kazakhstan
2. Turkmenistan
3. Uzbekistan

Select the correct answer using the codes below.


A. 1 only
B. 1 and 3 only
C. 2 and 3 only
D. 1 and 2 only

23
Online Prelims TEST - 5 (TEXTBOOK)
( InsightsIAS Mock Test Series for UPSC Preliminary Exam 2020 )

Correct Answer : A

Answer Justification :

Justification:

Q Source: Central Asia: Map based

32 The second most widely occurring particle in the Universe is

A. Photons
B. Neutrinos
C. Gelons
D. Gravitons

Correct Answer : B

Answer Justification :

Justification: Neutrinos, first proposed by Swiss scientist Wolfgang Pauli in 1930, are the second
most widely occurring particle in the universe, only second to photons, the particle which makes up
light. In fact, neutrinos are so abundant among us that every second, there are more than 100
trillion of them passing right through each of us — we never even notice them.

Neutrinos occur in three different types, or flavours. These are separated in terms of different
masses. From experiments so far, we know that neutrinos have a tiny mass, but the ordering of the
neutrino mass states is not known and is one of the key questions that remain unanswered till

24
Online Prelims TEST - 5 (TEXTBOOK)
( InsightsIAS Mock Test Series for UPSC Preliminary Exam 2020 )

today. This is a major challenge INO will set to resolve, thus completing our picture of the neutrino.

Neutrinos hold the key to several important and fundamental questions on the origin of the
Universe and the energy production in stars. Another important possible application of neutrinos is
in the area of neutrino tomograph of the earth, that is detailed investigation of the structure of the
Earth from core on wards. This is possible with neutrinos since they are the only particles which can
probe the deep interiors of the Earth.

Q Source:
https://www.insightsonindia.com/2019/07/12/insights-daily-current-affairs-pib-12-july-2019/

33 The applications of NAVIC, an independent regional navigation satellite system, includes


1. Weather forecasting
2. Disaster Management
3. Vehicle tracking and fleet management

Select the correct answer using the codes below.


A. 1 only
B. 2 and 3 only
C. 1 and 3 only
D. 1, 2 and 3

Correct Answer : B

Answer Justification :

Justification: Navigation with Indian Constellation (NavIC) is an independent regional navigation


satellite system designed to provide position information in the Indian region and 1500 km around
the Indian mainland.

IRNSS would provide two types of services, namely Standard Positioning Services available to all
users and Restricted Services provided to authorised users.

Its applications include:

Terrestrial, Aerial and Marine Navigation.


Disaster Management.
Vehicle tracking and fleet management.
Integration with mobile phones.
Precise Timing.
Mapping and Geodetic data capture.
Terrestrial navigation aid for hikers and travellers.
Visual and voice navigation for drivers.

Learning: It is a regional system and so its constellation will consist of seven satellites. Three of
these will be geostationary over the Indian Ocean, i.e., they will appear to be stationary in the sky
over the region, and four will be geosynchronous – appearing at the same point in the sky at the

25
Online Prelims TEST - 5 (TEXTBOOK)
( InsightsIAS Mock Test Series for UPSC Preliminary Exam 2020 )

same time every day. This configuration ensures each satellite is being tracked by at least one of
fourteen ground stations at any given point of time, with a high chance of most of them being visible
from any point in India.

Q Source:
https://www.insightsonindia.com/2019/07/18/insights-daily-current-affairs-pib-18-july-2019/

34 Chemically inactive nitrogen and argon gases are usually used to fill bulbs. This is because
1. They prolong the life of the bulb filament.
2. They significantly reduce the current-voltage fluctuation in the bulb.

Which of the above is/are correct?


A. 1 only
B. 2 only
C. Both 1 and 2
D. None

Correct Answer : A

Answer Justification :

Justification: The electric heating is also used to produce light, as in an electric bulb. Here, the
filament must retain as much of the heat generated as is possible, so that it gets very hot and emits
light. It must not melt at such high temperature.

A strong metal with high melting point such as tungsten (melting point 3380°C) is used for making
bulb filaments. The filament should be thermally isolated as much as possible, using insulating
support, etc.

The bulbs are usually filled with chemically inactive nitrogen and argon gases to prolong the life of
filament. Most of the power consumed by the filament appears as heat, but a small part of it is in
the form of light radiated.

Statement 2 is absurd.

Q Source: Page 218: Science Xth Standard NCERT

35 Which of these have been classified as Mauryan sites by historians?


1. Kaushambi
2. Girnar
3. Sopara

Select the correct answer using the codes below.


A. 1, 2 and 3
B. 1 and 2 only
C. 2 and 3 only

26
Online Prelims TEST - 5 (TEXTBOOK)
( InsightsIAS Mock Test Series for UPSC Preliminary Exam 2020 )

D. 1 only

Correct Answer : A

Answer Justification :

Justification: These are the major sites. We will cover some of these sites elsewhere in the tests
too. You don’t need to know so much about them.

Q Source: Ch2: Introduction to Indian arts: Class XI

36 The Ashgabat Agreement aims to


1. Build the shortest trade route between Central Asian countries and Iranian and Omani ports
2. Build a gas pipeline connecting all Central Asian and South Asian states

Select the correct answer using the codes below.


A. 1 only
B. 2 only
C. Both 1 and 2
D. None of the above

27
Online Prelims TEST - 5 (TEXTBOOK)
( InsightsIAS Mock Test Series for UPSC Preliminary Exam 2020 )

Correct Answer : A

Answer Justification :

Justification: The Ashgabat agreement is a Multimodal transport agreement between Oman, Iran,
Turkmenistan, Uzbekistan, Kazakhstan, and India for creating an international transport and transit
corridor facilitating transportation of goods between Central Asia and the Persian Gulf.

India formally joined Ashgabat agreement on 2nd February 2018. Ashgabat Agreement envisages
facilitation of transit and transportation of goods between Central Asia and the Persian Gulf.

Accession to the Agreement would diversify India’s connectivity options with Central Asia and have
a positive influence on India’s trade and commercial ties with the region

For enhanced connectivity, the Ashgabat agreement will also synchronize with the International
North–South Transport Corridor encompassing ship, rail including Trans-Caspian railway, and road
route for moving freight between India, Russia, Iran, Europe and Central Asia. The route primarily
involves moving freight from India, Iran, Azerbaijan and Russia via ship, rail and road.

Q Source: Map-based: Central Asia

http://mea.gov.in/press-releases.htm?dtl/29406/India_Joins_the_Ashgabat_Agreement

https://en.wikipedia.org/wiki/Ashgabat_agreement

37 The Indus Valley civilization Sites, among the following, that belong to India are
1. Harappa
2. Lothal
3. Dholavira
4. Mohenjodaro
5. Kalibangan

Select the correct answer using the codes below.


A. 1, 2 and 4 only
B. 3, 4 and 5 only
C. 2, 3 and 5 only
D. 1, 2, 3, 4 and 5

Correct Answer : C

Answer Justification :

Justification: While Harappa and Mohenjodaro are situated in Pakistan, the important sites
excavated in India are Lothal and Dholavira in Gujarat, Rakhigarhi in Haryana, Ropar in the Punjab,
Kalibangan and Balathal in Rajasthan.

28
Online Prelims TEST - 5 (TEXTBOOK)
( InsightsIAS Mock Test Series for UPSC Preliminary Exam 2020 )

Learning: Kalibangan has given the evidence of the earliest (2800 BC) ploughed agricultural field
ever revealed through an excavation as per Dr. B Lal. It is also a site which has given an evidence of
earliest recorded “Earthquake”.

Q Source: Page 9: Introduction to Indian Art – Class XI

38 Which of the following statements is correct with reference to the constitution of India?
1. It is a set of written rules that are accepted by all people living together in a country.
2. It specifies how the government will be constituted and the distribution of decision-making power
therein.
3. It expresses the aspirations of the people about creating a good society.

Select the correct answer using the codes below.


A. 1 only
B. 2 and 3 only
C. 1 and 3 only
D. 1, 2 and 3

Correct Answer : D

Answer Justification :

Justification: Constitution is the supreme law that determines the relationship among citizens and
also the relationship between the citizens and government. Moreover:

First, it generates a degree of trust and coordination that is necessary for different kind of
people to live together;
Second, it specifies how the government will be constituted, for e.g. by democratic elections.
Third, it lays down limits on the powers of the government and tells us what the rights of the
citizens are; and
Fourth, it expresses the aspirations of the people, for e.g. by preamble and DPSP.

Q source: Chapter 2: Democratic Politics: Based on concepts

39 The powers and the independence of the Indian judiciary allow it to act as the guardian of Indian
democracy. What are these powers?
1. Anyone can approach the higher judiciary if public interest is hurt by the actions of government.
2. Higher Judiciary can issue directives to check malpractices on the part of public officials.

Which of the above is/are correct?


A. 1 only
B. 2 only
C. Both 1 and 2
D. None

29
Online Prelims TEST - 5 (TEXTBOOK)
( InsightsIAS Mock Test Series for UPSC Preliminary Exam 2020 )

Correct Answer : C

Answer Justification :

Justification: Statement 1: This is called public interest litigation. The courts intervene to prevent
the misuse of the government’s power to make decisions. For e.g. the first PIL was filed on the poor
living condition of the jail inmates, the court took immediate action on the issue.

Statement 2: Courts can issue writs of Mandamus to warn public officials of lapse of duty and
unruly behaviour. They can also order them to perform certain functions, for e.g. ensuring
passenger safety on roads or safety of women at workplaces (Vishakha guidelines).
th
Q source: Revision: Chapter 5: Democratic Politics: Class 9 NCERT

40 India has officially declared itself free from which of the following diseases?
1. Chikungunya
2. Polio
3. Yaws
4. Avian Influenza
5. Leprosy
6. Smallpox

Select the correct answer using the codes below.


A. 2, 3 and 4 only
B. 1, 2 and 5 only
C. 2, 3, 4, 5 and 6 only
D. 1, 3, 4 and 6 only

Correct Answer : C

Answer Justification :

Justification: Statement 1: Many cases and deaths were recently reported in Delhi. India is not
free from the disease which is carried mainly by mosquitoes.

Statement 2 and 3: India has become the first country in the world to get the Yaws-free certificate
by the WHO.

This is the third major momentous public health feat for India, which was declared polio-free two
years ago, elimination of maternal and neonatal tetanus early this year and now becoming free from
Yaws, a chronic bacterial infection caused by Treponema pallidum subsp Pertenue.

Statement 4: India has officially declared itself free from highly contagious Avian Influenza (H5N1)
or bird flu and has notified the same to the World Organization for Animal Health (OIE).

Statement 5: Leprosy was officially declared to be eradicated in 2005, but many cases are still
found in India.

30
Online Prelims TEST - 5 (TEXTBOOK)
( InsightsIAS Mock Test Series for UPSC Preliminary Exam 2020 )

Statement 6: Smallpox was declared by WHO to be eradicated in 1979 from the World.

Q Source: Based on past year UPSC papers

41 Consider the following about the Vienna Convention for the Protection of the Ozone Layer.
1. It entered in force before Montreal protocol.
2. It includes legally binding reduction goals for the use of CFCs.
3. It has been ratified by all United Nations Member states.

Select the correct answer using the codes below.


A. 1 and 2 only
B. 1 and 3 only
C. 3 only
D. 1, 2 and 3

Correct Answer : B

Answer Justification :

Justification: Statement 2: It acts as a framework for the international efforts to protect the ozone
layer. These are laid out in the accompanying Montreal Protocol. However, it does not include
legally binding reduction goals for the use of CFCs, the main chemical agents causing ozone
depletion.

Statement 1 and 3: It was agreed upon at the 1985 Vienna Conference and entered into force in
1988 (Montreal in 1989). It is one of the most successful treaties of all time in terms of universality.
It has been ratified by 197 states (all UN members as well as the Niue, Holy See and the Cook
Islands) as well as European Union.

Learning: Montreal Protocol on Substances that Deplete targets phasing out production of
numerous Ozone Depleting Substances (ODSs).

Under it production and consumption of key ODSs like chlorofluorocarbon (CFCs), Methyl
Chloroform, CTC halons and Methyl Bromide have been phased-out globally.

Q Source: Frequently in news

42 Consider the following statements about Temple Architecture in India.


1. Vesara order was created through the selective mixing of Nagara and Dravida order.
2. Panchayatana style of architecture was practiced during Gupta period.

Select the correct answer using the codes below.


A. 1 only
B. 2 only
C. Both 1 and 2
D. None of the above

31
Online Prelims TEST - 5 (TEXTBOOK)
( InsightsIAS Mock Test Series for UPSC Preliminary Exam 2020 )

Correct Answer : C

Answer Justification :

Justification: Nagara is North Indian style; Dravida is South Indian. Vesara is a mixture of both.

A Hindu temple is a Panchayatana one when the main shrine is surrounded by four subsidiary
shrines. The origin of the name are the Sanskrit words Pancha (five) et ayatana (containing). It was
practiced during Gupta period.

Generally, the Hindu temple are built along a west-east axis. So the four subsidiary shrines are at
the north-east, south-east, south-west, north-west.

Temple at Deograh (Lalitpur, UP) is in the panchayatana style of architecture where the main shrine
is built on a rectangular plinth with four smaller subsidiary shrines at the four corners (making it a
total number of five shrines, hence the name, panchayatana).

Q Source: Chapter 6: Introduction to Indian Art – Class XI

43 Sachar Committee was setup with a focus on

A. Curbing violence and discrimination against Dalits


B. Social and economic development of Women in India
C. Social, economic, and educational conditions of the Muslim community of India
D. North-eastern India

Correct Answer : C

Answer Justification :

Learning: The committee was headed by former Chief Justice of the Delhi High Court Rajinder
Sachar and had six other members.

The report was the first of its kind to reveal the economic and social lag of the Muslim
community compared with other communities.
An issue highlighted was that while Muslims constitute 14% of the Indian population, they
only comprise 2.5% of the Indian bureaucracy.
The Sachar Committee concluded that the conditions facing Indian Muslims was below that of
Scheduled Castes and Scheduled Tribes.
In November 2013, the Gujarat government contended before the Supreme Court that the
Rajinder Sachar Committee was "unconstitutional," and that it only sought to help Muslims.
It has strongly criticized the manner in which the PMO set up the Sachar Committee in 2005
to survey the socio-economic conditions of Muslims, while "ignoring" other religious
minorities.

Q Source: Page 50: Democratic Politics: Class Xth NCERT

32
Online Prelims TEST - 5 (TEXTBOOK)
( InsightsIAS Mock Test Series for UPSC Preliminary Exam 2020 )

44 This gesture is also called renunciation mudra. The hand points downward away from the body as a
symbol for renunciation of secular pleasures:

A. Shramanamudra
B. Abhaya
C. Varada
D. Vitarka

Correct Answer : A

Answer Justification :

Learning: Option A: This gesture is also called renunciation mudra. The hand points downward
away from the body as a symbol for renunciation of secular pleasures.

Option B: This gesture is also called "Gesture of Fearlessness" or "blessing" or "fearless mudra".
Generally, this position is shown with the palms(s) facing outward and the fingers extended
upwards. This mudra is characteristic of Buddha Shakyamuni and Dhyani Buddha Amogasiddhi.

Option C: This gesture is also called "Gift bestowing Gesture of Compassion" or "conferring boon"
or "grace" mudra. The arm is extended all way down with palm facing outwards.

Option D: This Buddhist mudra is called "The Gesture of Debate" or "discussion" mudra

It is a mystic gesture of Taras and Bodhisattvas in which tips of thumb and index finger touch
forming a circle, and all the other fingers are extended upwards.

Vitarka Mudra:

Q Source: Improvisation: Page 104: Introduction to Indian Art – Class XI

45 Consider the following statements with respect to Hydrocarbon Exploration and Licensing Policy

33
Online Prelims TEST - 5 (TEXTBOOK)
( InsightsIAS Mock Test Series for UPSC Preliminary Exam 2020 )

1. provides uniform licences for exploration and production of all forms of hydrocarbons
2. offers hydrocarbon fields under a revenue-sharing model and gives marketing and pricing freedom
for the production of crude oil and natural gas

Select the correct answer using the codes below.


A. 1 only
B. 2 only
C. Both 1 and 2
D. None of the above

Correct Answer : C

Answer Justification :

Justification: Ministry of Petroleum and Natural Gas signs contracts for 32 blocks awarded under
Open Acreage Licensing Programme (OALP) Bid Rounds – II & III.

Hydrocarbon Exploration and Licensing Policy provides uniform licences for exploration and
production of all forms of hydrocarbons, enabling contractors to explore conventional as well as
unconventional oil and gas resources.

Fields are offered under a revenue-sharing model and throw up marketing and pricing freedom for
crude oil and natural gas produced.

Under the OALP, once an explorer selects areas after evaluating the National Data Repository
(NDR) and submits the EoI, it is to be put up for competitive bidding and the entity offering the
maximum share of oil and gas to the government is awarded the block.

NDR has been created to provide explorers’ data on the country’s repositories, allowing them to
choose fields according to their capabilities. Data received through the National Seismic
Programme, an in-depth study of 26 sedimentary basins, are continuously being added to the NDR.

Under the OALP, once an explorer selects areas after evaluating the National Data Repository
(NDR) and submits the EoI, it is to be put up for competitive bidding and the entity offering the
maximum share of oil and gas to the government is awarded the block.

NDR has been created to provide explorers’ data on the country’s repositories, allowing them to
choose fields according to their capabilities. Data received through the National Seismic
Programme, an in-depth study of 26 sedimentary basins, are continuously being added to the NDR.

Learning: The Hydrocarbon Exploration and Licensing Policy (HELP) replacing the erstwhile New
Exploration Licensing Policy (NELP) was approved in March 2016 and the Open Acreage Licensing
Programme (OALP) along with the National Data Repository (NDR) were launched in June 2017 as
the key drivers to accelerate the Exploration and Production (E&P) activities in India.

Q Source:
https://www.insightsonindia.com/2019/07/17/insights-daily-current-affairs-pib-17-july-2019/

34
Online Prelims TEST - 5 (TEXTBOOK)
( InsightsIAS Mock Test Series for UPSC Preliminary Exam 2020 )

46 Consider the following about the Nataraja dancing posture depicted in Chola period bronze
sculpture.
1. The dance is usually associated with the end of the cosmic world.
2. Tirobhava depicted in the sculpture shows the birth of the Universe.

Select the correct answer using the codes below.


A. 1 only
B. 2 only
C. Both 1 and 2
D. None of the above

Correct Answer : A

Answer Justification :

Justification: In the bronze sculpture Shiva has been shown balancing himself on his “right” leg
and suppressing the apasmara, the demon of ignorance or forgetfulness, with the foot of the same
leg. At the same time he raises his left leg in bhujangatrasita stance, which represents tirobhava,
that is kicking away the veil of “maya or illusion” from the devotee’s mind.

His four arms are outstretched and the main right hand is posed in abhaya hasta or the gesture
suggesting. The upper right holds the damaru his favourite musical instrument to keep on the beat
tala.

35
Online Prelims TEST - 5 (TEXTBOOK)
( InsightsIAS Mock Test Series for UPSC Preliminary Exam 2020 )

Q Source: Page 107: Introduction to Indian Art – Class XI

47 Biogas is largely made up of

A. Nitric Oxide
B. Hydrogen
C. Methane
D. Carbon dioxide

Correct Answer : C

Answer Justification :

Learning: Bio-gas is an excellent fuel as it contains up to 75% methane. It burns without smoke,
leaves no residue like ash in wood, charcoal and coal burning.

Its heating capacity is high and generates gases like methane, carbon dioxide, hydrogen and
hydrogen sulphide during heating.

India is rich in biomass and has a potential of 16,881 MW (agro-residues and plantations), 5000 MW
(bagasse cogeneration) and 2700 MW (energy recovery from waste).

Q Source: Page 247: Science Xth Standard NCERT

48 Consider the following statements.


Assertion (A): Continuous passage of electric current in a circuit produces heat.
Reason (R): Electrons are consumed in an electric circuit.

In the context of the above, which of these is correct?


A. A is correct, and R is an appropriate explanation of A.
B. A is correct, but R is not an appropriate explanation of A.
C. A is correct, but R is incorrect.
D. A is incorrect, but R is correct.

Correct Answer : C

Answer Justification :

Justification: Many people think that electrons are consumed in an electric circuit. This is wrong.

We pay the electricity board or electric company to provide energy to move electrons through the
electric gadgets like electric bulb, fan and engines, not to consume or destroy electrons.

We pay for the energy that we use, not the number of electrons we consume. So, R is clearly wrong.

Q Source: Page 219: Science Xth Standard NCERT

36
Online Prelims TEST - 5 (TEXTBOOK)
( InsightsIAS Mock Test Series for UPSC Preliminary Exam 2020 )

49 Consider the following statements.


Assertion (A): The sky appears dark to passengers flying at very high altitudes.
Reason (R): Light scattering is not prominent at such heights.

In the context of the above, which of these is correct?


A. A is correct, and R is an appropriate explanation of A.
B. A is correct, but R is not an appropriate explanation of A.
C. A is correct, but R is incorrect.
D. Both A and R are incorrect.

Correct Answer : A

Answer Justification :

Justification: The molecules of air and other fine particles in the atmosphere have size smaller
than the wavelength of visible light. These are more effective in scattering light of shorter
wavelengths at the blue end than light of longer wavelengths at the red end.

The red light has a wavelength about 1.8 times greater than blue light. Thus, when sunlight passes
through the atmosphere, the fine particles in air scatter the blue colour (shorter wavelengths) more
strongly than red.

The scattered blue light enters our eyes. If the earth had no atmosphere, there would not have been
any scattering. Then, the sky would have looked dark. The sky appears dark to passengers flying at
very high altitudes, as scattering is not prominent at such heights.

Q Source: Page 196: Science Xth Standard NCERT

50 Phamsana, Shekhari and Valabhi are

A. prominent schools of Buddhist Tibetan Tangka paintings


B. pre-historic caves made during the Pleistocene age.
C. They are styles of temple architecture of Northern India
D. None of the above

Correct Answer : C

Answer Justification :

Learning: The main styles of North Indian temples include Latina, Phamsana, Shekhari and Valabhi
types.

Valabhi style of temples comprises ayatakara talachhanda yojana (rectangular ground plan), a
doorway to one of its longer parasvas (side) and a semi- cylindrical sikhara (superstructure).
About fifty percent of Valabhi style temples are in Uttarakhand.
Latina types are mainly used for housing the garbhagriha.

37
Online Prelims TEST - 5 (TEXTBOOK)
( InsightsIAS Mock Test Series for UPSC Preliminary Exam 2020 )

Phamsana type shikhara are broader and shorter than Latina type.
In many north Indian temples, the phamsana type is used for mandapas while the main
garbhagriha is housed in a Latina building.

Q Source: AR: Chapter 6: Introduction to Indian Art – Class XI

51 The major international protocols that address genetically modified organisms (GMOs) is/are
1. Helsinki Convention
2. Nagoya-Kuala Lumpur Supplementary Protocol
3. Geneva Protocol
4. Cartagena protocol

Select the correct answer using the codes below.


A. 1 and 2 only
B. 2 and 4 only
C. 1 and 3 only
D. 1, 2 and 4 only

Correct Answer : B

Answer Justification :

Justification: Statement 4: India is a party to the United Nations Convention on Biological


Diversity signed at Rio de Janeiro.

The Cartagena Protocol on Biosafety (CPB), the first international regulatory framework for safe
transfer, handling and use of Living Modified Organisms (LMOs) was negotiated under the aegis of
the Convention on Biological Diversity (CBD).

Statement 2: Following years of negotiations over the question of liability for GMO-produced
damages, in 2010, the Nagoya-Kuala Lumpur Supplementary Protocol on Liability and Redress to
the Cartagena Protocol on Biosafety (the Supplementary Protocol) was adopted.

India has already ratified the protocol in 2014, you can see here
http://pib.nic.in/newsite/PrintRelease.aspx?relid=110927

Statement 1: It is the Convention on the Protection of the Marine Environment of the Baltic Sea
Area.

Statement 3: Geneva Protocol addresses “Prohibition of the Use in War of Asphyxiating, Poisonous
or other Gases and of Bacteriological Methods of Warfare”.

Q Source: http://www.moef.nic.in/sites/default/files/treaties/international-treaties.html

52 The Intergovernmental Panel on Climate Change (IPCC) is the leading international body for the
assessment of climate change. Consider the following about it.
1. Its membership is open to all member countries of the United Nations (UN).

38
Online Prelims TEST - 5 (TEXTBOOK)
( InsightsIAS Mock Test Series for UPSC Preliminary Exam 2020 )

2. It was established by the United Nations Framework Convention on Climate Change (UNFCCC).
3. Scientists from all over the world contribute to the work of the IPCC on a voluntary basis without
receiving any pecuniary benefits.
4. It publishes “Assessments Reports” on an annual basis addressing climate change related issues.

Select the correct answer using the codes below.


A. 1 and 3 only
B. 2 and 4 only
C. 4 only
D. 1, 2, 3 and 4

Correct Answer : A

Answer Justification :

Justification: Statement 1: As an intergovernmental body, membership of the IPCC is open to all


member countries of the United Nations (UN) and WMO. Currently 195 countries are Members of
the IPCC. Governments participate in the review process and the plenary Sessions.

Statement 2: It was established by the United Nations Environment Programme (UNEP) and the
World Meteorological Organization (WMO) in 1988 to provide the world with a clear scientific view
on the current state of knowledge in climate change and its potential environmental and socio-
economic impacts.

Statement 3: The Intergovernmental Panel on Climate Change is a huge and yet very small
organization. Thousands of scientists from all over the world contribute to the work of the IPCC on
a voluntary basis as authors, contributors and reviewers. None of them is paid by the IPCC. The
work of the IPCC is guided by a set of principles and procedures.

Statement 4: The first report was published in 1990; second in 1995; third in 2001; fourth in 2007;
and Fifth very recently in 2013-14.

These are published materials composed of the full scientific and technical assessment of climate
change.

Q Source: AR: http://www.moef.nic.in/sites/default/files/treaties/international-treaties.html + Often


in news

53 Aflatoxins in mycotoxins affect nearly one-fourth of world's food crops They are produced by?

A. Fungi
B. Bacteria
C. Yeast
D. Protozoa

Correct Answer : A

39
Online Prelims TEST - 5 (TEXTBOOK)
( InsightsIAS Mock Test Series for UPSC Preliminary Exam 2020 )

Answer Justification :

Justification: Food and Agriculture Organization (FAO) of the United Nations estimates that 25%
of the food crops are affected by mycotoxins, the most notorious of which is alfatoxins.

Molds are microscopic fungi that live on plant or animal matter.

Mycotoxins are poisonous substances produced by certain molds found primarily in grain and
nut crops, but are also known to be on celery, grape juice, apples, and other produce.
Mycotoxins can penetrate into the parts of food that are not visibly mouldy as well. It is
therefore necessary to throw away all of the food if any part of it is mouldy. They are also
notoriously difficult to destroy as they are stable to both heat and chemicals.

Learning: Aflatoxin is (poison) any of a family of mycotoxins, produced by molds of the aspergillus
genus.

Aflatoxins have been associated with various diseases, such as aflatoxicosis in livestock,
domestic animals, and humans throughout the world.
Patulin is a mycotoxin produced by a variety of molds, in particular, Aspergillus and
Penicillium and Byssochlamys. Most commonly found in rotting apples, in general the amount
of patulin in apple products is viewed as a measure of the quality of the apples used in
production.
Aspergillus is used in the commercial production of citric acid.

Q Source: Revision: Previous test syllabus

54 Consider the following statements.


Assertion (A): Viruses found in food cannot infect humans.
Reason (R): Unlike Bacteria, Viruses cannot multiply in foods.

In the context of the above, which of these is correct?


A. A is correct, and R is an appropriate explanation of A
B. A is correct, but R is not an appropriate explanation of A.
C. A is correct, but R is incorrect.
D. A is incorrect, but R is correct.

Correct Answer : D

Answer Justification :

Justification: Viruses in food do affect humans, but Viruses need live host for multiplication. They
can only multiply inside living cells of other organisms. They do not multiply in food items, so food
only acts as a mechanism of transmission.

The main modes of transmission of Virus are by food handlers and the use of dirty utensils, which
transfer the virus to food whereupon it is ingested by humans.

Q Source: Revision: Previous test syllabus

40
Online Prelims TEST - 5 (TEXTBOOK)
( InsightsIAS Mock Test Series for UPSC Preliminary Exam 2020 )

55 The government has announced its plans to raise a portion of its gross borrowing from overseas
markets. Why is this decision important for the Indian economy?
1. It will expand the availability of domestic funds to the private sector for borrowing.
2. It will help curb government’s budgetary deficit.

Select the correct answer using the codes below.


A. 1 only
B. 2 only
C. Both 1 and 2
D. None of the above

Correct Answer : A

Answer Justification :

Justification: Overseas bond issue means a government bond or sovereign bond is a form of debt
that the government undertakes wherein it issues bonds with the promise to pay periodic interest
payments and also repay the entire face value of the bond on the maturity date. So far, the
government has only issued bonds in the domestic market.

Statement 1: The government has been arguing that the quantum of its borrowing within India is
‘crowding out’ the private sector. In other words, it is saying that government borrowing is at such
a level that there are not enough funds available for the private sector to adequately meet its credit
and investment needs.

If the private sector cannot borrow adequately, then it cannot invest as it wants to, and that cripples
one major engine of economic growth.

Therefore, borrowing overseas allows the government to raise funds in such a way that there is
enough domestic credit available for the private sector.

Statement 2: The government may try to reduce fiscal deficit by overseas bond, since they may be
offered at cheaper rates of interest, but fiscal deficit is not the same as budgetary deficit.

These are two different terms: budgetary deficit and fiscal deficit – budgetary deficit is basically
total government spending – total government receipts. Fiscal deficit is total government spending –
total government revenue. Government receipts include both revenue and loan arrangements. So, a
budgetary deficit is always zero since total expenditure cannot exceed total receipts, but a fiscal
deficit is a non-zero quantity since the government needs to borrow in order to bridge the fiscal
deficit.

Learning: However, India is not likely to be viewed as a risky proposition by the international
market and so is likely to fetch an attractive rate for the bonds. Cheap and plentiful funds, however,
should not encourage the government to borrow too heavily from abroad.

This would also lead to a quicker increase to its foreign exchange reserves, which would lead to a
stronger rupee at a time when it is already appreciating against the dollar. A stronger rupee would
encourage imports at a time when the government is trying to curb them, and discourage exports at

41
Online Prelims TEST - 5 (TEXTBOOK)
( InsightsIAS Mock Test Series for UPSC Preliminary Exam 2020 )

a time when they are being encouraged.

Q Source:
https://www.insightsonindia.com/2019/07/15/insights-daily-current-affairs-pib-15-july-2019/

56 Litmus solution is a purple dye, which is extracted from “X”. “X” is a plant belonging to the division
Thallophyta, and is commonly used as an indicator. X here is

A. Lichens
B. Mosses
C. Shell
D. Rose

Correct Answer : A

Answer Justification :

Learning: Litmus is a water soluble mixture of different dyes extracted from lichens. It is often
absorbed onto filter paper to produce one of the oldest forms of pH indicator, used to test materials
for acidity.

When the litmus solution is neither acidic nor basic, its colour is purple.

There are many other natural materials like red cabbage leaves, turmeric, coloured petals of some
flowers such as Hydrangea, Petunia and Geranium, which indicate the presence of acid or base in a
solution.

Q Source: Page 17: Science Xth Standard NCERT

57 Which of the following human body fluids is the most acidic in nature?

A. Blood
B. Gastric Juice
C. Saliva
D. Tears

Correct Answer : B

Answer Justification :

Learning:

42
Online Prelims TEST - 5 (TEXTBOOK)
( InsightsIAS Mock Test Series for UPSC Preliminary Exam 2020 )

Q Source: Page 26: Science Xth Standard NCERT

58 As per the Global MPI 2019 Report prepared by the United Nations Development Programme (UNDP)
and the Oxford Poverty and Human Development Initiative, in India
1. Poverty decreased the least amongst Scheduled Tribes, as compared to other ethnic or religious
communities, over 2005-06 to 2015-16
2. Bihar remains the poorest state in 2015-16, with more than half of its population living in poverty
3. Incidence of multidimensional poverty declined only marginally between 2005-06 and 2015-16, even
as the general income levels of the population increased

Select the correct answer using the codes below.


A. 2 and 3 only
B. 2 only
C. 1 and 3 only
D. 3 only

Correct Answer : B

Answer Justification :

Justification: Multi-dimensional poverty defines poor not only on the basis of income, but on other
indicators, including poor health, poor quality of work and the threat of violence.

Key findings: India specific:

Incidence of multidimensional poverty almost halved between 2005-06 and 2015-16, climbing
down to 27.5%, indicating that the number of poor people in India fell by more than 271
million within ten years.

43
Online Prelims TEST - 5 (TEXTBOOK)
( InsightsIAS Mock Test Series for UPSC Preliminary Exam 2020 )

Incidence of multidimensional poverty halved in India due to faster progress among the
poorest in the country. Among states, Jharkhand had the greatest improvement, with
Arunachal Pradesh, Bihar, Chhattisgarh, and Nagaland only slightly behind.
However, Bihar was still the poorest state in 2015- 16, with more than half of its population
living in poverty. In 2015-16, the four poorest states – Bihar, Jharkhand, Uttar Pradesh, and
Madhya Pradesh – were home to 196 million multidimensional poor people – over half of all
the people living in multidimensional poverty in India.
Least poor regions also saw reduction in poverty. Relative to their starting levels, they netted
some of the highest rates of reduction. For example, Kerala, one of the least poor regions in
2006, reduced its MPI by around 92%.
The positive trend of pro-poor poverty reduction was seen also across religions and caste
groups. In both cases, the poorest groups (Muslims and Scheduled Tribes) reduced poverty
the most over the ten years from 2005-06 to 2015-16.
The poorest district is Alirajpur in Madhya Pradesh, where 76.5% of people are poor – the
same as Sierra Leone in Sub-Saharan Africa. Only eight countries have higher rates of MPI.

Q Source:
https://www.insightsonindia.com/2019/07/13/insights-daily-current-affairs-pib-13-july-2019/

59 The scheme ‘Paramarsh’ is an initiative of

A. University Grants Commission (UGC)


B. NITI Aayog
C. Ministry of Statistics and Programme Implementation
D. Lok Sabha Secretariat

Correct Answer : A

Answer Justification :

Learning: Union Ministry of Human Resource Development has launched ‘Paramarsh’ – a


University Grants Commission (UGC) scheme.

The scheme is for Mentoring National Accreditation and Assessment Council (NAAC) Accreditation
Aspirant Institutions to promote Quality Assurance in Higher Education.

The scheme will be a paradigm shift in the concept of mentoring of institution by another well
performing institution to upgrade their academic performance and enable them to get accredited by
focusing in the area of curricular aspects, teaching-learning & evaluation, research, innovation,
institutional values & practices etc.

The Scheme will be operationalized through a “Hub & Spoke” model wherein the Mentor
Institution, called the “Hub” is centralized and will have the responsibility of guiding the Mentee
institution through the secondary branches the “Spoke” through the services provided to the
mentee for self improvement.

Q Source:

44
Online Prelims TEST - 5 (TEXTBOOK)
( InsightsIAS Mock Test Series for UPSC Preliminary Exam 2020 )

https://www.insightsonindia.com/2019/07/19/insights-daily-current-affairs-pib-19-july-2019/

60 What are the general applications of carbon Nanotubes?


1. They can be used to reinforce graphite in tennis rackets
2. Their structure allows them to be used as a container for transporting a drug in the body
3. They are used as semiconductors in electrical circuits

Select the correct answer using the codes below.


A. 1 and 3 only
B. 2 only
C. 1 and 2 only
D. 1, 2 and 3

Correct Answer : D

Answer Justification :

Justification: The fullerenes are a large class of allotropes of carbon and are made of balls, ‘cages’
or tubes of carbon atoms.

Statement 1 and 3: Nanotubes are cylindrical fullerenes. Their unique molecular structure results in
extraordinary macroscopic properties, including high tensile strength, high electrical conductivity,
high ductility, high heat conductivity, and relative chemical inactivity.

Nanotubes can be used to reinforce graphite in tennis rackets because they are very strong. Due to
the above mentioned properties, they are also used as semiconductors in electrical circuits.

Statement 2: The nanotube's structure allows it to be used as a container for transporting a drug in
the body. A molecule of the drug can be placed inside the nanotube cage. This keeps the drug
'wrapped up' until it reaches the site where it is needed. In this way, a dose that might be damaging
to other parts of the body can be delivered safely to, for example, a tumour.

Q Source: AR: Page 61: Science Xth Standard NCERT

61 Internal and Extra Budgetary Resources (IEBR) is sometimes seen in news. What is IBER?

A. Monetization of deficit through Forex Reserves


B. Basel III Capitalization of Commercial Banks
C. Financing received from IFC Masala bonds
D. Resources raised by the PSUs through profits, loans and equity.

Correct Answer : D

Answer Justification :

45
Online Prelims TEST - 5 (TEXTBOOK)
( InsightsIAS Mock Test Series for UPSC Preliminary Exam 2020 )

Learning: IEBR is an important part of the Central plan of the Government of India and constitutes
the resources raised by the PSUs through profits, loans and equity.

The Union Cabinet has recently given approval to service principal and interest against the
Extra Budgetary Resources (EBR) of Rs. 16,300 crore by Union Government for infrastructure
spending.
The EBR is proposed to finance funds to be raised by Indian Renewable Energy Development
Agency (IREDA), Power Finance Corporation (PFC), Inland Waterways Authority of India
(IWAI), and National Bank for Agriculture and Rural Development (NABARD).
This implies that interest and principal in respect of the EBR of Rs.16,300 crore will be
financed by Union Government by making suitable budget provisions.
It will supplement the efforts of the Union Government to improve infrastructure spending
and also improve revenue-capital mix of expenditure for a more sustainable growth.

Q Source: http://pib.nic.in/newsite/PrintRelease.aspx?relid=150969

62 Newlands’ Law of Octaves is related to

A. Composition of Music
B. Classification of elements
C. Changing phases of matter
D. Distribution of mass in the Solar system

Correct Answer : B

Answer Justification :

Note: We are venturing into topics/areas hitherto untouched by conventional thinking, and by
UPSC too. You will find many questions that may not seem very important from exam point of view;
nevertheless we should be covering them because in UPSC, there are no trends. You should be
prepared for anything.

Justification: John Newlands, an English scientist, arranged the then known elements in the order
of increasing atomic masses. He started with the element having the lowest atomic mass (hydrogen)
and ended at thorium which was the 56th element.

He found that every eighth element had properties similar to that of the first. He compared this to
the octaves found in music.

Therefore, he called it the ‘Law of Octaves’. It is known as ‘Newlands’ Law of Octaves’. In


Newlands’ Octaves, the properties of lithium and sodium were found to be the same. Sodium is the
eighth element after lithium. Similarly, beryllium and magnesium resemble each other.

Q Source: Page 80: Science Xth Standard NCERT

63 Which of the following is/are plant hormones?

46
Online Prelims TEST - 5 (TEXTBOOK)
( InsightsIAS Mock Test Series for UPSC Preliminary Exam 2020 )

1. Thyroxin
2. Gibrellin
3. Cytokinin
4. Oestrogen

Select the correct answer using the codes below.


A. 1 and 4 only
B. 2 and 3 only
C. 3 only
D. 1, 2, 3 and 4

Correct Answer : B

Answer Justification :

Justification: Statement 2: Gibberellins (GAs) are plant hormones that regulate growth and
influence various developmental processes, including stem elongation, germination, dormancy,
flowering, sex expression, enzyme induction, and leaf and fruit senescence.

Statement 3: They promote cell division, or cytokinesis, in plant roots and shoots.

Learning: Abscisic acid is one example of a hormone which inhibits growth. Its effects include
wilting of leaves.

Auxin also promotes plant growth.

Q Source: Page 122: Science Xth Standard NCERT

64 With reference to Kisan Urja Suraksha evam Utthaan Mahabhiyan (KUSUM), consider the following
statements.
1. KUSUM aims to provide energy sufficiency and sustainable irrigation access to farmers.
2. It would give farmers an option to sell additional power to the grid through solar power projects set
up on their barren lands.

Which of the above is/are correct?


A. 1 only
B. 2 only
C. Both 1 and 2
D. None

Correct Answer : C

Answer Justification :

Justification: Earlier this year, the Cabinet approved the Kisan Urja Suraksha evam Utthaan
Mahabhiyan (KUSUM).

47
Online Prelims TEST - 5 (TEXTBOOK)
( InsightsIAS Mock Test Series for UPSC Preliminary Exam 2020 )

With a Budget allocation of Rs.34,000 crore, and a similar contribution expected from the States,
KUSUM aims to provide energy sufficiency and sustainable irrigation access to farmers.

KUSUM proposes a 60% subsidy for the pumps, borne equally by the Centre and the States, and the
remaining 40% will be the farmer’s contribution, 10% as down payment and 30% through loans.

It would provide extra income to farmers, by giving them an option to sell additional power to the
grid through solar power projects set up on their barren lands. It would help in de-dieselising the
sector as also the DISCOMS.

At present, despite burgeoning farm power subsidies, nearly 30 million farmers, especially
marginal landholders, use expensive diesel for their irrigation needs as they have no access to
electricity.
More than half of India’s net sownarea remains unirrigated.
First, KUSUM should aim to reduce the existing disparity among States with regard to solar
pumps deployment and irrigation access:
Chhattisgarh and Rajasthan together account for about half of the two lakh solar pumps
currently deployed in the country.
This is surprising given the low irrigation demand in the former and poor groundwater
situation in the latter.
On the other hand, States such as Bihar, Uttar Pradesh and West Bengal, where penetration
of diesel pumps is among the highest, have not managed to deploy any significant number of
solar pumps.
This disparity highlights poor State budget allocation towards solar pumps and the lack of
initiative by State nodal agencies.
To encourage more equitable deployment of 5 lakh offgrid pumps by 2022, the Centre should
incentivise States through target linked financial assistance, and create avenues for peer
learning.

Q Source: https://www.insightsonindia.com/2019/07/20/insights-into-editorial-rethinking-kusum/

65 Power of a spherical lens is mainly dependent upon its


1. Length
2. Width
3. Focal length
4. Clarity

Select the correct answer using the codes below.


A. 2 and 4 only
B. 1, 2 and 4 only
C. 3 only
D. 1, 2, 3 and 4

Correct Answer : C

Answer Justification :

48
Online Prelims TEST - 5 (TEXTBOOK)
( InsightsIAS Mock Test Series for UPSC Preliminary Exam 2020 )

Justification: Two lenses with same focal length can have different lengths, width and clarity; yet
their power will remain the same. So, 1, 2 and 4 are incorrect.

Power of lens is the ability of the lens to converge the rays of light falling on it. Definition: The
power of a lens is the reciprocal of its focal length in meters. When, f = 1 m, P = 1/f = 1/1 = 1
dioptre. 1 dioptre is the power of a lens whose focal length is 1 meter.

Q Source: Page 184: Science Xth Standard NCERT

66 Consider the following statements.


Assertion (A): The apparent position of the star on earth is slightly different from its actual position.
Reason (R): The starlight, on entering the earth’s atmosphere, undergoes refraction continuously
before it reaches the earth.

In the context of the above, which of these is correct?


A. A is correct, and R is an appropriate explanation of A.
B. A is correct, but R is not an appropriate explanation of A.
C. A is correct, but R is incorrect.
D. Both A and R are incorrect.

Correct Answer : A

Answer Justification :

Justification: The atmospheric refraction occurs in a medium of gradually changing refractive


index. Since the atmosphere bends starlight towards the normal, the apparent position of the star is
slightly different from its actual position.

The star appears slightly higher (above) than its actual position when viewed near the
horizon.
Further, this apparent position of the star is not stationary, but keeps on changing slightly,
since the physical conditions of the earth’s atmosphere are not stationary.
Since the stars are very distant, they approximate point-sized sources of light.
As the path of rays of light coming from the star goes on varying slightly, the apparent
position of the star fluctuates and the amount of starlight entering the eye flickers – the star
sometimes appears brighter, and at some other time, fainter, which is the twinkling effect

Q Source: Page 195: Science Xth Standard NCERT

67 The International Court of Justice (ICJ) has which of these primary functions
1. To settle legal disputes submitted by States in accordance with established international laws
2. To suo moto recognize incidences of gross human rights violations by sovereign governments and
move the concerned case to Hague

Select the correct answer using the codes below.


A. 1 only

49
Online Prelims TEST - 5 (TEXTBOOK)
( InsightsIAS Mock Test Series for UPSC Preliminary Exam 2020 )

B. 2 only
C. Both 1 and 2
D. None of the above

Correct Answer : A

Answer Justification :

Justification: Established in 1946 to replace the Permanent Court of International Justice, the ICJ
mainly operates under the statute of its predecessor, which is included in the UN Charter.

It has two primary functions: to settle legal disputes submitted by States in accordance with
established international laws, and to act as an advisory board on issues submitted to it by
authorized international organizations.

Context: International Court of Justice (ICJ) has directed Pakistan to review conviction order of
Kulbhushan Jadhav and, until then, put his death sentence on hold. ICJ has also asked Pakistan to
allow India consular access at earliest.

Key observations made by the ICJ:

Islamabad has violated Article 36 of Vienna Convention of Consular Relations, 1963, by not
informing India about Jadhav’s arrest immediately after Pakistan Army had taken him into custody.

India had been deprived of ‘right to communicate with and have access to Jadhav, to visit him in
detention and to arrange for his legal representation’.

Q Source:
https://www.insightsonindia.com/2019/07/18/insights-daily-current-affairs-pib-18-july-2019/

68 In medieval India, Lapis lazuli was often used in ornaments and official seals. The stone was largely
sourced from which part of the World?

A. Sri Lanka
B. Sumatra
C. China
D. Northern Afghanistan

Correct Answer : D

Answer Justification :

Learning: It is a rich blue semi-precious stone sometimes flecked with gold.

The main source in the ancient world was the mountains of Badakshan, northern Afghanistan, from
where it was traded widely. Lapis lazuli was used as inlay in ornaments, jewellery, seals, etc

50
Online Prelims TEST - 5 (TEXTBOOK)
( InsightsIAS Mock Test Series for UPSC Preliminary Exam 2020 )

Today mines in northeast Afghanistan and Pakistan are still the major source of lapis lazuli.

Important amounts are also produced from mines west of Lake Baikal in Russia, and in the Andes
mountains in Chile. Smaller quantities are mined in Italy, Mongolia, the United States and Canada.

It was used by some of the most important artists of the Renaissance and Baroque, including
Masaccio, Perugino, Titian and Vermeer,

Q Source: Glossary: Introduction to Indian Art – Class XI

69 Low-temperature thermal desalination (LTTD) technique works on the principle that


1. Water at a certain depth in the ocean is colder than surface water.
2. There is no variation in salinity between the deeper layers and surface layers of ocean water.

Which of the above is/are correct?


A. 1 only
B. 2 only
C. Both 1 and 2
D. None

Correct Answer : A

Answer Justification :

Justification: Statement 1: Water in the ocean 1,000 or 2,000 feet below is about 4º C to 8º C
colder than surface water. So, salty surface water is collected in a tank and subject to high pressure
(via an external power source). This pressured water vapourises and this is trapped in tubes or a
chamber. Cold water plumbed from the ocean depths is passed over these tubes and the vapour
condenses into fresh water and the resulting salt diverted away.

Statement 2: This is wrong, there is a difference in salinity in these layers.

Learning: Ocean Thermal Energy Conversion: It will draw power from the vapour generated as a
part of the desalination process. This vapour will run a turbine and thereby will be independent of
an external power source. While great in theory, there is no guarantee it will work commercially.
For one, this ocean-based plant requires a pipe that needs to travel 50 kilometres underground in
the sea before it reaches the mainland.

Q Source: Based on ideas of salination covered in previous tests

70 An interesting feature of medieval India was the sarais which ringed cities and dotted a vast space of
the Indian subcontinent. Sarais were
1. Local private godowns which provided grain storage facility to adjoining farms
2. Dargahs that hosted religious and spiritual discussions

Select the correct answer using the codes below.

51
Online Prelims TEST - 5 (TEXTBOOK)
( InsightsIAS Mock Test Series for UPSC Preliminary Exam 2020 )

A. 1 only
B. 2 only
C. Both 1 and 2
D. None of the above

Correct Answer : D

Answer Justification :

Learning: Sarais were largely built on a simple square or rectangular plan and were meant to
provide temporary accommodation to Indian and foreign travellers, pilgrims, merchants, traders,
etc.

In effect, sarais were public domains which thronged with people of varied cultural backgrounds.
This led to cross-cultural interaction, influences and syncretic tendencies in the cultural mores of
the times and at the level of the people.

Q Source: Improvisation: Page 115: Introduction to Indian Art – Class XI

71 Which of these countries in South and South-east Asia have some of their beaches received Blue Flag
Certification?
1. India
2. China
3. South Korea

Select the correct answer using the codes below.


A. 2 only
B. 3 only
C. 1, 2 and 3
D. 1 and 3 only

Correct Answer : C

Answer Justification :

Justification:

The Blue Flag Programme for beaches and marinas is run by the international, non-governmental,
non-profit organisation FEE (the Foundation for Environmental Education).

It started in France in 1985 and has been implemented in Europe since 1987, and in areas outside
Europe since 2001, when South Africa joined. Spain tops the list with 566 such beaches; Greece
and France follow with 515 and 395, respectively.

A Number of beaches from India have also been selected for the Blue Flag Certification, for e.g.
Odisha's Chandrabhaga beach, and other beaches in A&N, Goa, Pudducherry, Maharashtra,

52
Online Prelims TEST - 5 (TEXTBOOK)
( InsightsIAS Mock Test Series for UPSC Preliminary Exam 2020 )

Lakshadweep etc. See here


https://timesofindia.indiatimes.com/world/chandrabhaga-beach-gets-blue-flag-tag-heres-what-it-mea
ns/articleshow/64473078.cms?from=mdr

There are nearly 33 criteria that must be met to qualify for a Blue Flag certification, such as the
water meeting certain quality standards, having waste disposal facilities, being disabled- friendly,
have first aid equipment, and no access to pets in the main areas of the beach. Some criteria are
voluntary and some compulsory.

Q Source:
https://www.insightsonindia.com/2019/07/16/insights-daily-current-affairs-pib-16-july-2019/

72 In India, the largest contributor to energy generation is

A. Coal-based power plants


B. Nuclear plants
C. Natural gas based plants
D. Hydel power plants

Correct Answer : A

Answer Justification :

Learning: India became the world's third largest producer of electricity in the year 2013 with 4.8%
global share in electricity generation surpassing Japan and Russia.

India's electricity sector consumes about 72% of the coal produced in the country. The high ash
content in India's coal affects the thermal power plant's potential emissions.

Q Source: Page 244: Science Xth Standard NCERT

53
Online Prelims TEST - 5 (TEXTBOOK)
( InsightsIAS Mock Test Series for UPSC Preliminary Exam 2020 )

73 Which of the following most appropriately describes “Toranas”?


#00000

A. Gateways to Buddhist Stupas


B. Collection of Chinars and Pietra duras seen in decorative works in Islami architecture
C. Style of dome-based architecture introduced by Turks
D. Calligraphy inscribed on interior walls of royal durbars

Correct Answer : A

Answer Justification :

Learning: Usually made of stone, Toranas mark the entrance to a Buddhist shrine or stupa or to a
Hindu temple.

Toranas typically consist of two pillars carrying two or three transverse beams that extend beyond
the pillars on either side.

Strongly similar to wooden construction, toranas are often covered from top to bottom with
exquisite sculpture.

The four toranas of the Great Stupa at Sanchi are relevant examples.

Q Source: AR: Page 111: Introduction to Indian Art – Class XI

74 National Social Assistance Programme (NSAP) does not cover which of the following types of
schemes?

A. Housing
B. Food
C. Disability pension
D. Support to widowed women

Correct Answer : A

Answer Justification :

Learning: The NSAP is a Centrally Sponsored Scheme under the Ministry of Rural Development. It
came into effect from 15th August,1995.

Presently NSAP comprises of five schemes, namely:

Indira Gandhi National Old Age Pension Scheme (IGNOAPS).


Indira Gandhi National Widow Pension Scheme (IGNWPS).
Indira Gandhi National Disability Pension Scheme (IGNDPS).
National Family Benefit Scheme NFBS).

54
Online Prelims TEST - 5 (TEXTBOOK)
( InsightsIAS Mock Test Series for UPSC Preliminary Exam 2020 )

Annapurna (food to BPL)

It represents a significant step towards the fulfillment of the DPSP in Article 41 of the Constitution.
In particular, Article 41 of the Constitution of India directs the State to provide public assistance to
its citizens in case of unemployment, old age, sickness and disablement and in other cases of
undeserved want within the limit of its economic capacity and development.

It aims to provide financial assistance to the elderly, widows and persons with disabilities in the
form of social pensions.

Coverage: It currently covers more than three crore people who are below the poverty line (BPL),
including about 80 lakh widows, 10 lakh disabled and 2.2 crore elderly.

Q Source:
https://www.insightsonindia.com/2019/07/18/insights-daily-current-affairs-pib-18-july-2019/

75 The Sun is visible to us about two minutes before the actual sunrise, and about two minutes after the
actual sunset because of

A. Atmospheric refraction
B. Constructive interference
C. Dispersion of sunlight
D. Total Internal Reflection

Correct Answer : A

Answer Justification :

Learning: By actual sunrise, we mean the actual crossing of the horizon by the Sun.

Figure below shows the actual and apparent positions of the Sun with respect to the horizon.

The time difference between actual sunset and the apparent sunset is about 2 minutes. The
apparent flattening of the Sun’s disc at sunrise and sunset is also due to the atmospheric refraction
phenomenon.

55
Online Prelims TEST - 5 (TEXTBOOK)
( InsightsIAS Mock Test Series for UPSC Preliminary Exam 2020 )

Q Source: Page 195: Science Xth Standard NCERT

76 In the “Tribhanga” posture depicted in many famous ancient sculptures

A. Body stands gracefully with weight placed on one leg in a meditative posture
B. The dancer turns his head towards her peers in alternate clockwise and anti-clockwise fashion
C. Dancer performs on a metal plate which touches one-third of his feet at any given time
D. Head is inclined to one side while the upper and lower body take opposite directions to each
other

Correct Answer : D

Answer Justification :

Learning: The ideal postures of the body in movement are based upon these Bhangas or bend
which represent the change of the body from the central straight line or balance of the figure.

Bhangas are of 4 kinds:

Abhanga (slight flexion)


Samabhanga (equipose)
Atibhanga (excessive flexion)
Tribhanga (three flexions)

Tribhanga posture is the thrice-bent figure in which the head is inclined to one side, the upper body
is bent in the opposite direction and part of the body below the waist takes again the reverse
direction. The Tribhanga actions are dramatic energetic muscle actions.

The ‘Dancing Girl’ in tribhanga posture from Mohenjodaro is the earliest bronze sculpture datable
to 2500 BCE.

Q Source: AR: Page 103: Introduction to Indian Art – Class XI

56
Online Prelims TEST - 5 (TEXTBOOK)
( InsightsIAS Mock Test Series for UPSC Preliminary Exam 2020 )

77 If State laws on subjects mentioned in the Concurrent List conflict with a Central Law, which of these
follows?
1. The state law prevails over the Central law if the State legislature passes it again with special
majority.
2. In most cases, the matter is moved to the Supreme Court which decides on the validity of the
respective laws in the larger national interest.

Which of the above is/are correct?


A. 1 only
B. 2 only
C. Both 1 and 2
D. None of the above

Correct Answer : D

Answer Justification :

Justification: The Constitution provides a scheme for demarcation of powers through three ‘lists’
in the seventh schedule. But, the Constitution also provides primacy to Parliament on concurrent
list items: if there is a conflict, a central law will override a state law, i.e. the provisions of the state
law will not take effect.

There is no provision of the intervention of either the SC or the Governor in case this is an ordinary
matter. If there are constitutional issues involved in the passing of a law, the Supreme court may
participate, but the only to ensure the validity of the law, and not to say that the state law reigns
over the Central law (which would be unconstitutional).

Q Source: Page 17: Democratic Politics: Class Xth NCERT

78 Consider the following statements about local self-government in India.


1. It is constitutionally mandatory to hold regular elections to local government bodies.
2. An independent institution, State Election Commission, has been created in each State to conduct
panchayat and municipal elections.

Which of the above is/are correct?


A. 1 only
B. 2 only
C. Both 1 and 2
D. None

Correct Answer : C

Answer Justification :

Learning: Seats are reserved in the elected bodies and the executive heads of these institutions for
the Scheduled Castes Scheduled Tribes and Other Backward Classes (OBC provision can be made
57
Online Prelims TEST - 5 (TEXTBOOK)
( InsightsIAS Mock Test Series for UPSC Preliminary Exam 2020 )

by State legislatures; it is not compulsory). At least one-third of all positions are reserved for
women.

The State governments are required to share some powers and revenue with local government
bodies. The nature of sharing varies from State to State.

Q Source: Democratic Politics: Xth NCERT

79 Which of the following famous images/sculpture are likely to be found in Ajanta caves?
1. Padmapani and Vajrapani
2. Shiva slaying Andhaka and Wedding of Shiva
3. Mahaparinirvana of Buddha
4. Trimurti, Gangadhara and Ardhanarishvara

Select the correct answer using the codes below.


A. 2 and 3 only
B. 1, 2 and 4 only
C. 1 and 3 only
D. 1, 2, 3 and 4

Correct Answer : C

Answer Justification :

Justification: Statement 1: Seated Buddha in Dharmachakrapravartana mudra is notable sculpture


while the notable paintings include Padmapani and Vajrapani.

Statement 3: Mahaparinirvana of Buddha on the right aisle wall and the assault of Mara during
Buddha’s penance adorns the same wall. The Mahaparinirvana of the Buddha is when he finally
achieves release from the mortal world.

Statement 2 and 4: These are found in Elephanta caves. Described as a "masterpiece of Gupta-
Chalukyan art", the most important sculpture in the Elephanta caves is the Trimurti. The carved
panel facing this one is a two-level depiction of Ravana lifting Kailash.

Q Source: AR: Page 37-39: Introduction to Indian Art – Class XI

80 Kihoto Hollohan case sometimes seen in news concerns with

A. Violation of labour norms at Workplace


B. Anti-defection law
C. Judicial overreach by a certain High Court
D. Emergency health support

Correct Answer : B

58
Online Prelims TEST - 5 (TEXTBOOK)
( InsightsIAS Mock Test Series for UPSC Preliminary Exam 2020 )

Answer Justification :

Justification: In the arguments in the Supreme Court in the case related to the political crisis in
Karnataka, advocate has cited the landmark judgment in Kihoto Hollohan vs Zachillhu And Others
(1992), in which the court upheld the sweeping discretion available to the Speaker in deciding cases
of disqualification of MLAs.

What was the Kihoto Hollohan case?

The law covering the disqualification of legislators and the powers of the Speaker in deciding such
matters became part of the statute book in 1985 when the Tenth Schedule to the Constitution was
adopted which is related to the matter of anti detection.

A constitutional challenge to the Tenth Schedule was settled by the apex court in Kihoto Hollohan.

The principal question before the Supreme Court in the case was whether the powerful role given to
the Speaker violated the doctrine of Basic Structure — the judicial principle that certain basic
features of the Constitution cannot be altered by amendments by Parliament, laid down in the
landmark judgment in Kesavananda Bharati vs State Of Kerala (1973).

What is the extent of the Speaker’s powers?

Paragraph 6(1) of the Tenth Schedule describes the Speaker’s sweeping discretionary powers: “If
any question arises as to whether a member of a House has become subject to disqualification
under this Schedule, the question shall be referred for the decision of the Chairman or, as the case
may be, the Speaker of such House and his decision shall be final.”

Q Source:
https://www.insightsonindia.com/2019/07/18/insights-daily-current-affairs-pib-18-july-2019/

81 Which of the following are legally mandatory conditions for political parties in or political candidates
fighting elections in India?
1. Giving a certain percentage of election tickets to women candidates
2. A record of the adherence of a political party to its own Constitution
3. Submit an affidavit giving details of his property and criminal cases pending against him to the
concerned authority

Select the correct answer using the codes below.


A. 1 and 2 only
B. 2 only
C. 3 only
D. 1 and 3 only

Correct Answer : C

Answer Justification :

Justification: These are some of the compulsory provisions:

59
Online Prelims TEST - 5 (TEXTBOOK)
( InsightsIAS Mock Test Series for UPSC Preliminary Exam 2020 )

The Constitution was amended to prevent elected MLAs and MPs from changing parties.
The Supreme Court passed an order to reduce the influence of money and criminals. Now, it
is mandatory for every candidate who contests elections to file an affidavit giving details of his
property and criminal cases pending against him.
The Election Commission passed an order making it necessary for political parties to hold
their organisational elections and file their income tax returns

Besides these, many suggestions are often made to reform political parties:

A law should be made to regulate the internal affairs of political parties. It should be made
compulsory for political parties to maintain a register of its members, to follow its own
constitution, to have an independent authority, to act as a judge in case of party disputes, to
hold open elections to the highest posts.
It should be made mandatory for political parties to give a minimum number of tickets, about
one-third, to women candidates. Similarly, there should be a quota for women in the decision
making bodies of the party.

Q Source: Page 85-86: Democratic Politics: Class Xth NCERT

82 “Totipotency”, that is useful in Plant Tissue Culture, means that


1. the plant has native reproductive abilities of plants via pollination mechanism
2. there is an absence of cell walls to facilitate gene transfer mechanism

Which of the above is/are correct?


A. 1 only
B. 2 only
C. Both 1 and 2
D. None of the above

Correct Answer : D

Answer Justification :

Learning: Plant tissue culture is a collection of techniques used to maintain or grow plant cells,
tissues or organs under sterile conditions on a nutrient culture medium of known composition
Totipotency is the capacity to generate whole plant from any cell/ explant. Plant tissue culture is
widely used to produce clones of a plant in a method known as micropropagation.

Single cells, plant cells without cell walls (protoplasts), pieces of leaves, stems or roots can
often be used to generate a new plant on culture media given the required nutrients and plant
hormones.

Different techniques in plant tissue culture may offer certain advantages over traditional methods of
propagation, including:

60
Online Prelims TEST - 5 (TEXTBOOK)
( InsightsIAS Mock Test Series for UPSC Preliminary Exam 2020 )

The production of exact copies of plants that produce particularly good flowers, fruits, or have
other desirable traits.
To quickly produce mature plants.
The production of multiples of plants in the absence of seeds or necessary pollinators to
produce seeds.
The regeneration of whole plants from plant cells that have been genetically modified etc.

Q Source: Page 132: Science Xth Standard NCERT

83 Which of the following vision-related advantages occur to human beings for having two eyes, as
against only one eye?
1. Reduced Depth of field
2. Enhanced Width of field

Which of the above is/are correct?


A. 1 only
B. 2 only
C. Both 1 and 2
D. None

Correct Answer : B

Answer Justification :

Justification: S1: It actually increases depth of field.

The range of object distances for which the circles of confusion are so small that the image is sharp
enough to be considered 'in focus' is called the depth of field.

The range of image distances over which the image of an improperly focused object is acceptably
sharp is called the depth of focus.

An eye cannot focus on objects that are at different distances from it at the same time. Therefore
strictly speaking once the eye has accomodated (adjusted the strength of the eye-lens) to view an
objact at an object distance O from it, it should only be able to see objacts that are that precise
distance away in focus. However, if the object is almost that distance away the object forms an
image which is acceptably sharp enough for the barain of the observer to make out enough detail
for him/her to class it as in focus.

The depth of field deals with the range of 'acceptable' object distances that can be classed as 'in
focus' when it is accomodated to focus on a particular spot.

S2: There are several other advantages of our having two eyes Instead of one.

It gives a wider field of view. A human being has a horizontal field of view of about 150
degrees with one eye and of about 180 degrees with two eyes. The ability to detect faint
objects is thus enhanced with two detectors instead of one.

61
Online Prelims TEST - 5 (TEXTBOOK)
( InsightsIAS Mock Test Series for UPSC Preliminary Exam 2020 )

Some animals, usually prey animals, have their two eyes positioned on opposite sides of their
heads to give the widest possible field of view.
Shut one eye and the world looks flat two-dimensional. Keep both eyes open and the world
takes on the third dimension of depth. Because our eyes are separated by a few centimetres
each eye sees a slightly different image.
Our brain combines the two images into one, using the extra information to tell us how close
or far away things are.

Q Source: Chapter 11: Science Xth Standard NCERT

84 The Roaring Forties are

A. Strong westerly winds found in the Southern Hemisphere


B. Feeble easterly winds found in the Northern Hemisphere
C. Thunderstorms often found near the equator
D. Equatorial counter-currents that move near to the eastern coastal shores in lower latitudes

Correct Answer : A

Answer Justification :

Learning: They are generally found between the latitudes of 40 and 50 degrees.

The strong west-to-east air currents are caused by the combination of air being displaced
from the Equator towards the South Pole and the Earth's rotation, and there are few
landmasses to serve as windbreaks.
The Roaring Forties were a major aid to ships sailing the Brouwer Route from Europe to the
East Indies or Australasia during the Age of Sail, and in modern usage are favoured by
yachtsmen on round-the-world voyages and competitions.
The boundaries of the Roaring Forties are not consistent, and shift north or south depending
on the season.
Similar but stronger conditions occur in more southerly latitudes and are referred to as the
Furious Fifties and Shrieking or Screaming Sixties.

Q Source: AR: Past year UPSC papers

85 Which of the following will violate the General Anti Avoidance Rules (GAAR) in India?
1. Any speculative activities in the security markets
2. Any real estate deal that has been settled in a short period of time

Which of the above is/are correct?


A. 1 only
B. 2 only
C. Both 1 and 2
D. None

62
Online Prelims TEST - 5 (TEXTBOOK)
( InsightsIAS Mock Test Series for UPSC Preliminary Exam 2020 )

Correct Answer : D

Answer Justification :

Justification: About GAAR: GAAR provisions are targeted at arrangement or transactions made
specifically to avoid taxes.

Evolution: In 2007, Vodafone entered the Indian market by buying Hutchison Essar. The deal took
place in Cayman Islands. The Indian government claimed over US$2 billion were lost in taxes.

Later, a notice was sent to Vodafone. Vodafone claimed that the transaction was not taxable as it
was between two foreign firms. The government claimed that the deal was taxable as the underlying
assets involved were located in India. Then came the provisions of GAAR in India.

Provisions: The regulation allows tax officials to deny tax benefits, if a deal is found without any
commercial purpose other than tax avoidance.

It allows tax officials to target participatory notes.


Under GAAR, the investor has to prove that the participatory note was not set to avoid taxes.
It also allows officials to deny double taxation avoidance benefits, if deals made in tax havens
like Mauritius were found to be avoiding taxes

Statement 1: Therefore, clearly it does not apply to all such transactions. Only if the speculation was
done in order to specifically avoid taxes, it will come under the radar of GAAR.

Statement 2: The same applies to real estate transactions. The time period of sale/purchase is not
the issue; it is the purpose with which it is done matters.

Q Source: AR: Past year UPSC papers

86 With reference to the Large Hadron Collider (LHC), consider the following statements.
1. The composite particles inside LHC composed of quarks are held together by gravitational force
alone.
2. Cryogenic stage temperature is maintained inside the accelerator to allow for the collision of high
energy particles using superconductor.

Select the correct answer using the codes below.


A. 1 only
B. 2 only
C. Both 1 and 2
D. Neither 1 nor 2

Correct Answer : B

Answer Justification :

63
Online Prelims TEST - 5 (TEXTBOOK)
( InsightsIAS Mock Test Series for UPSC Preliminary Exam 2020 )

Justification: Statement 1: These quarks cannot be held together by such a weak gravitational
force. It is the “Strong Force” (electromagnetic) which holds them together.

Statement 2: Inside the accelerator, two high-energy particle beams travel at close to the speed of
light before they are made to collide. The beams travel in opposite directions in separate beam
pipes – two tubes kept at ultrahigh vacuum.

They are guided around the accelerator ring by a strong magnetic field maintained by
superconducting electromagnets.

The electromagnets are built from coils of special electric cable that operates in a superconducting
state, efficiently conducting electricity without resistance or loss of energy. This requires chilling
the magnets to ‑271.3°C – a temperature colder than outer space!

Q Source: AR: past year UPSC papers

87 The Central Asian countries bordering Caspian Sea are


1. Tajikistan
2. Uzbekistan
3. Turkmenistan

Select the correct answer using the codes below.


A. 1 and 2 only
B. 3 only
C. 2 and 3 only
D. 1 only

Correct Answer : B

Answer Justification :

Justification:

64
Online Prelims TEST - 5 (TEXTBOOK)
( InsightsIAS Mock Test Series for UPSC Preliminary Exam 2020 )

Q Source: Central Asia: Map based

88 With reference to art and archaeological history of India, which of the following are associated with
the reign of Ashoka?
1. Rock-cut Elephant at Dhauli
2. Lomus Rishi caves
3. Badami Caves
4. Udayagiri Caves

Select the correct answer using the codes below.


A. 1 and 2 only
B. 3 and 4 only
C. 1 and 3 only
D. 2 and 4 only

Correct Answer : A

Answer Justification :

Justification: Statement 1: Rock-cut Elephant at Dhauli was created during Ashoka’s reign
(272-231 BC). It is the earliest Buddhist sculpture of Odisha.

Statement 2: Located in the Barabar and Nagarjuni hills of Jehanabad district in Bihar, this rock-cut
cave was carved out as a sanctuary. It was built during the Ashokan period of the Maurya Empire in
the 3rd century BC, as part of the sacred architecture of the Ajivikas.

Statement 3: The Badami cave temples are a complex of Hindu and Jain cave temples located in
Bagalkot district in northern part of Karnataka, India. The caves are considered an example of
Indian rock-cut architecture, especially Badami Chalukya architecture, which dates from the 6th
century.

Statement 4: It is a Hindu cave and was made during the Gupta reign. It contains the famous
Varaha sculpture.

Q Source: AR: Page 21: Introduction to Indian Art – Class XI

89 Election in a country is necessarily democratic, if


1. A large number of voters have registered for the poll.
2. A government controlled body supervises election process.

Select the correct answer using the codes below.


A. 1 only
B. 2 only
C. Both 1 and 2
D. None of the above

65
Online Prelims TEST - 5 (TEXTBOOK)
( InsightsIAS Mock Test Series for UPSC Preliminary Exam 2020 )

Correct Answer : D

Answer Justification :

Justification: Statement 1: For e.g. Iraq may have a large voter base, many alleged that its 2003
elections were not democratic.

Statement 2: The election conducting body needs to be independent, and not controlled by the
government.

It is not enough that elections be based on rules and procedures. There are many requirements for
an election to be called as democratic.

Learning: These include:

All parties must have a fair chance of winning or losing


Election body should be independent.
The losing party should accept the verdict
There should be universal adult franchise so that political equality is ensured.

Q source: Chapter 4: Democratic Politics NCERT: Class 9th

90 The Dharmachakrapravartana Sutta


1. the main topic of this sutra is the Four Noble Truths
2. contains the Buddhist concepts of the Middle Way, impermanence, and dependent origination

Select the correct answer using the codes below.


A. 1 only
B. 2 only
C. Both 1 and 2
D. None of the above

Correct Answer : C

Answer Justification :

Learning: The Setting in Motion of the Wheel of the Dharma Sutra or Promulgation of the Law
Sutra is considered to be a record of the first teaching given by Gautama Buddha after he attained
enlightenment.

According to tradition, the Buddha gave this teaching in Sarnath, India, to the "five ascetics",
his former companions with whom he had spent six years practicing austerities.
The main topic of this sutra is the Four Noble Truths, which are the central teachings of
Buddhism that provide a unifying theme, or conceptual framework, for all of Buddhist
thought.
This sutra also introduces the Buddhist concepts of the Middle Way, impermanence, and

66
Online Prelims TEST - 5 (TEXTBOOK)
( InsightsIAS Mock Test Series for UPSC Preliminary Exam 2020 )

dependent origination.

Q Source: Page 20: Introduction to Indian Art – Class XI

91 The Harappans procured materials for craft production from various sites. Match the following sites
with the availability of the material/stone in that site.
1. Lothal: Carnelian
2. Shortughai: Lapis Lazuli
3. Khetri: Copper

Select the correct matches using the codes below.


A. 1 and 2 only
B. 1, 2 and 3
C. 2 and 3 only
D. 1 and 3 only

Correct Answer : B

Answer Justification :

Justification: The Harappans procured materials for craft production in various ways. For
instance, they established settlements such as Nageshwar and Balakot in areas where shell was
available.

Other such sites were Shortughai, in far-off Afghanistan, near the best source of lapis lazuli, a blue
stone that was apparently very highly valued, and Lothal which was near sources of carnelian (from
Bharuch in Gujarat), steatite (from south Rajasthan and north Gujarat) and metal (from Rajasthan).

Another strategy for procuring raw materials may have been to send expeditions to areas such as
the Khetri region of Rajasthan (for copper) and south India (for gold). These expeditions established
communication with local communities. Occasional finds of Harappan artefacts such as steatite
micro beads in these areas are indications of such contact.

There is evidence in the Khetri area for what archaeologists call the Ganeshwar-Jodhpura culture,
with its distinctive non-Harappan pottery and an unusual wealth of copper objects. It is possible
that the inhabitants of this region supplied copper to the Harappans.

92 Democratic world governance will necessarily be achieved if


1. All nations in the world turn democratic
2. All nations in the world join United Nations

Which of the above is/are correct?


A. 1 only
B. 2 only
C. Both 1 and 2
D. None

67
Online Prelims TEST - 5 (TEXTBOOK)
( InsightsIAS Mock Test Series for UPSC Preliminary Exam 2020 )

Correct Answer : D

Answer Justification :

Justification: Statement 1: The yardstick for Democratic world governance is whether each of the
countries has free and equal say in the decisions that affect them. It depends more on the
International institutional architecture in the UN, than in democracy being brought in the nations of
the world.

For e.g. UNSC does not need the approval of UNGA even though the non-permanent members are
elected by UNGA. The permanent members wield a lot of power in UNSC.

Statement 2: Only few countries (e.g. Abkhazia due to limited recognition) are not members of UN,
and their joining the UN would not make a difference in the decision-making structure.

Q Source: Revision: Previous tests syllabus

93 In international law, a condominium, sometimes seen in news

A. Is a refugee territory over from which no nation has the right to expel the protectorates
B. Is an economic entity that is free to enter into or exit from agreements made independently
with sovereign governments
C. Is an international organization that has been granted observer status in any of the United
Nations Agencies or arms
D. Is a political territory in or over which multiple sovereign powers formally agree to share
equal dominium and rights

Correct Answer : D

Answer Justification :

Learning: One example is Antarctica which is a condominium governed by the Antarctica Treaty
system.

States can exercise their rights jointly, without dividing it into 'national' zones.

A major problem, and the reason so few of such territories have existed, is the difficulty of ensuring
co-operation between the sovereign powers; once the understanding fails, the status is likely to
become untenable.

Q Source: Issues based on Antarctica

https://metro.co.uk/2017/12/11/owns-antarctica-british-antarctic-territory-7149257/

94 With reference to Swadesh Darshan Scheme, consider the following statements.


1. It intends to develop theme-based tourist circuits in the country.

68
Online Prelims TEST - 5 (TEXTBOOK)
( InsightsIAS Mock Test Series for UPSC Preliminary Exam 2020 )

2. The scheme is 100% centrally funded for the project components undertaken for public funding.
3. One the implied aims of the scheme would be to showcase tribal culture and art to enhance
livelihood opportunities for tribal populations in the country.

Select the correct answer using the codes below.


A. 3 only
B. 1 and 3 only
C. 1, 2 and 3
D. 2 and 3 only

Correct Answer : C

Answer Justification :

Justification: Statement 1 and 3: The Ministry of Tourism, under the Swadesh Darshan Scheme,
has identified tribal circuit as one of the fifteen thematic circuits for development of tourism
infrastructure in the country with the objective of showcasing tribal culture, art, handicrafts and
providing livelihood and enhance employment opportunities for tribal populations in the country.

Objective: to develop theme-based tourist circuits in the country. These tourist circuits will be
developed on the principles of high tourist value, competitiveness and sustainability in an
integrated manner.

Statement 2: Features of Swadesh Darshan Scheme:

The scheme is 100% centrally funded for the project components undertaken for public
funding.
To leverage the voluntary funding available for Corporate Social Responsibility
(CSR)initiatives of Central Public Sector Undertakings and corporate sector.
Funding of individual project will vary from state to state and will be finalised on the basis of
detailed project reports prepared by PMC (Programme Management Consultant).
A National Steering Committee (NSC)will be constituted with Minister in charge of M/O
Tourism as Chairman, to steer the mission objectives and vision of the scheme.
A Mission Directorate headed by the Member Secretary, NSC as a nodal officer will help in
identification of projects in consultation with the States/ UTs governments and other stake
holders.
PMC will be a national level consultant to be appointed by the Mission Directorate.

Q Source:
https://www.insightsonindia.com/2019/07/17/insights-daily-current-affairs-pib-17-july-2019/

95 Which of these factors make it difficult to implement a single power tariff nationwide?
1. Not all states are endowed with sufficient natural resources to procure these resources and produce
power locally and economically
2. Different geographical terrains of different states allow different means of production of power, the
costs of which may vary widely.

69
Online Prelims TEST - 5 (TEXTBOOK)
( InsightsIAS Mock Test Series for UPSC Preliminary Exam 2020 )

Select the correct answer using the codes below.


A. 1 only
B. 2 only
C. Both 1 and 2
D. None of the above

Correct Answer : C

Answer Justification :

Justification: The State Electricity Regulatory Commissions (SERCs) decide on the power tariff
after utilities file their cost of power with the regulators. Because power tariffs entail a number of
state-specific factors, a uniform nationwide tariff is a proposition that would be difficult to
implement.

States such as Jharkhand or Odisha or Chhattisgarh, which have had coal-fired thermal capacity,
would typically have lower tariffs because of the base-load capacities that they possess.

States such as Himachal or Uttarakhand would have low tariffs because of hydropower capacities
that are either fully or partially depreciated, and from which the home state, under the Ministry of
Power’s Tariff Policy, gets access to 40 per cent of the power free of cost.

A state like Gujarat, which has capacities based on imported coal, will have comparatively higher
tariffs. States such as Delhi or Punjab, which buy power from outside to meet domestic
requirements either through long-term power purchase agreements (PPAs) or spot power purchases
at the power exchanges (PXs), have higher tariffs, as their power mix has high-cost power.

Q Source:
https://www.insightsonindia.com/2019/07/16/insights-daily-current-affairs-pib-16-july-2019/

96 Consider the following statements.


1. ‘Crimes Against Humanity (CAH)’ are dealt with under the Rome Statute of the International
Criminal Court (ICC).
2. India is neither a party to the Rome Statute or to Genocide Convention (1948).

Select the correct answer using the codes below.


A. 1 only
B. 2 only
C. Both 1 and 2
D. None of the above

Correct Answer : A

Answer Justification :

Justification: Statement 1: Internationally, CAH are dealt with under the Rome Statute of the

70
Online Prelims TEST - 5 (TEXTBOOK)
( InsightsIAS Mock Test Series for UPSC Preliminary Exam 2020 )

International Criminal Court (ICC).

They are defined as offences such as murder, extermination, enslavement, deportation, torture,
imprisonment and rape committed as a part of “widespread or systematic attack directed against
any civilian population, with knowledge of the attack”.

India is not a party to the Rome Statute, which means that it is under no obligation at present to
enact a separate legislation dealing with CAH.

Even after ratification of the Genocide Convention (1948), India has not enacted it in domestic
legislation.

Statement 2: The Convention on the Prevention and Punishment of the Crime of Genocide
(Genocide Convention) is an instrument of international law that codified for the first time the
crime of genocide.
According to the Genocide Convention, genocide is a crime that can take place both in time of
war as well as in time of peace.
The Genocide Convention was the first human rights treaty adopted by the General Assembly
of the United Nations on 9 December 1948 and signified the international community’s
commitment to ‘never again’ after the atrocities committed during the Second World War.

Q Source:
https://www.insightsonindia.com/2019/07/13/insights-into-editorial-crimes-that-indias-statute-books-
have-failed-to-define/

97 Which of the following is NOT an example of ‘sunrise’ industry?

A. Food processing Industry


B. Wellness and Hospitality
C. Renewable energy industry
D. Automobile industry

Correct Answer : D

Answer Justification :

Justification: A sunrise industry is typically characterized by high growth rates, numerous start-
ups and an abundance of venture capital funding.

Sunrise industries generally have plenty of "buzz" surrounding them as public awareness about the
sector increases and investors get attracted to its long-term growth prospects.

Automobile industry is a well-established industry, but renewable energy industry holds a lot of
promise, which is why D is the answer.

Q Source: Revision: Page 54: 8th NCERT Geography

71
Online Prelims TEST - 5 (TEXTBOOK)
( InsightsIAS Mock Test Series for UPSC Preliminary Exam 2020 )

98 A line joining Tashkent to Ashbagat will NOT cut through which of these nations?

A. Kazakhstan
B. Turkmenistan
C. Kyrgyzstan
D. Uzbekistan

Correct Answer : C

Answer Justification :

Learning: It cuts through a small part of Kazakhstan, but not Kyrgyzstan.

99 In 2018, the United Nations (UN) issued a Global Call to Action to elevate the importance of and
prioritize action on water, sanitation and hygiene (WASH) in all health care facilities. Why is WASH
important?
1. It can improve maternal, neonatal and child health.
2. It can prevent unnecessary use of antibiotics, thereby helping contain antimicrobial resistance.

Select the correct answer using the codes below.


A. 1 only
B. 2 only
C. Both 1 and 2
D. None of the above

72
Online Prelims TEST - 5 (TEXTBOOK)
( InsightsIAS Mock Test Series for UPSC Preliminary Exam 2020 )

Correct Answer : C

Answer Justification :

Justification: The WHO/UNICEF JMP report, WASH in Health Care Facilities, is the first
comprehensive global assessment of water, sanitation and hygiene (WASH) in health care facilities.

It outlines that, WASH services in many facilities across the world are missing or substandard.

It finds that 1 in 8 health care facilities has no water service and 1 in 5 has no sanitation service –
impacting close to 900 million and more than 1.5 billion people, respectively.

One in every six healthcare facilities was estimated to have no hygiene service (meaning it lacked
hand hygiene facilities at points of care, as well as soap and water at toilets), while data on waste
management and environmental cleaning was inadequate across the board.

Statement 1 and 2: When a healthcare facility lacks adequate WASH services, infection prevention
and control are severely compromised. This has the potential to make patients and health workers
sick from avoidable infections.

As a result (and in addition), efforts to improve maternal, neonatal and child health are undermined.

Lack of WASH facilities also results in unnecessary use of antibiotics, thereby spreading
antimicrobial resistance.

In 2018, the United Nations (UN) issued a Global Call to Action to elevate the importance of and
prioritize action on WASH in all health care facilities, including primary, secondary and tertiary
facilities in both the public and private sectors.

The call recognises the important role WASH plays in preventing infections, saving lives, and
improving quality of care.

Q Source:
https://www.insightsonindia.com/2019/07/19/insights-into-editorial-a-wash-for-healthcare/

100 Aqua regia is


1. a very strong alkali
2. a mixture of Magnesium Hydroxide and Sodium Bicarbonate

Select the correct answer using the codes below.


A. 1 only
B. 2 only
C. Both 1 and 2
D. None of the above

Correct Answer : D

73
Online Prelims TEST - 5 (TEXTBOOK)
( InsightsIAS Mock Test Series for UPSC Preliminary Exam 2020 )

Answer Justification :

Learning: It is one of the few reagents that is able to dissolve gold and platinum. It is strongly
acidic.

It is made by mixing one part nitric acid and three parts hydrochloric acid.

The acid was named by alchemists because it can dissolve the noble metals gold and
platinum. Tantalum, iridium and a few other metals are not dissolved by it.
It is also used to clean certain lab machines from tiny metal particles. It is particularly used in
the purification and extraction of gold and platinum.
It has an interesting history to it.
When Germany invaded Denmark in World War II, the Hungarian chemist George de Hevesy
took the Nobel Prize medals of Max von Laue and James Franck and dissolved them in Aqua
regia so that Germans cannot take these medals.
Later he submitted the dissolved Gold to the Royal Swedish Academy which again made
medals for the Nobel laureates from the gold.

Q Source: Page 44: Science Xth Standard NCERT

74

S-ar putea să vă placă și